Sie sind auf Seite 1von 130

More Than 2000 Solved MCQs on Research Methodology

&
Short Questions Answers
1- Hypothesis refers to
A. The outcome of an experiment
B. A conclusion drawn from an experiment
C. A form of bias in which the subject tries to outguess the experimenter
D. A tentative statement about the relationship
2- Statistics is used by researchers to
A. Analyze the empirical data collected in a study
B. Make their findings sound better
C. Operationally define their variables
D. Ensure the study comes out the way it was intended
3- A literature review requires
A. Planning
B. Good & clear writing
C. Lot of rewriting
D. All of the above
4- A literature review is based on the assumption that
A. Copy from the work of others
B. Knowledge accumulates and learns from the work of others
C. Knowledge dis-accumulates
D. None of the above option
5- A theoretical framework
A. Elaborates the r/s among the variables
B. Explains the logic underlying these r/s
C. Describes the nature and direction of the r/s
D. All of the above

6- Which of the following statement is not true?


A. A research proposal is a document that presents a plan for a project
B. A research proposal shows that the researcher is capable of successfully conducting
the proposed research project
C. A research proposal is an unorganized and unplanned project
D. A research proposal is just like a research report and written before the research
project
7- Preliminary data collection is a part of the
Research Method (STA 630) Success Objectives
A. Descriptive research
B. Exploratory research
C. Applied research
D. Explanatory research
8- Conducting surveys is the most common method of generating
A. Primary data
B. Secondary data
C. Qualitative data
D. None of the above
9- After identifying the important variables and establishing the logical reasoning in
theoretical framework, the next step in the research process is
A. To conduct surveys
B. To generate the hypothesis
C. To focus group discussions
D. To use experiments in an investigation
10- The appropriate analytical technique is determined by
A. The research design
B. Nature of the data collected
C. Nature of the hypothesis
D. Both A & B

11- Personal interviews conducted in shopping malls are known as:


a. Mall interviews
b. Mall intercept interviews
c. Brief interviews
d. None of the given options
12-WATS lines provided by long distance telephone service at fixed rates. In this
regard,
WATS is the abbreviation of:
e. West Africa Theological Seminary
f. Washtenaw Area Transportation Study
g. Wide Area Telecommunications Service
h. World Air Transport Statistics
13-A list of questions which is handed over to the respondent, who reads the
questions
and records the answers himself is known as the:
i. Interview schedule
j. Questionnaire
k. Interview guide
l. All of the given options
14-One of the most critical stages in the survey research process is:
m. Research design
n. Questionnaire design
o. Interview design
p. Survey design
Research Method (STA 630) Success Objectives
15-Question that consists of two or more questions joined together is called a:
q. Double barreled question

r. General question
s. Accurate question
t. Confusing question

16-The number of questionnaires returned or completed divided by the total


number of
eligible people who were contacted or asked to participate in the survey is called the:
u. Response rate
v. Participation rate
w. Inflation rate
x. None of the given options
17-To obtain the freest opinion of the respondent, when we ask general question
before a
specific question then this procedure is called as the:
y. Research technique
z. Qualitative technique
aa. Funnel technique
bb. Quantitative technique
18-A small scale trial run of a particular component is known as:
cc. Pilot testing
dd. Pre-testing
ee. Lab experiments
ff. Both A & B
18-Field testing of the questionnaire shows that:
gg. Respondents are willing to co-operate
hh. Respondents are not willing to co-operate
ii. Respondents do not like any participation
jj. All of the given options
19- Service evaluation of hotels and restaurants can be done by the:

kk. Self-administered questionnaires


ll. Office assistant
mm. Manager
nn. None of the given options

20- Service evaluation of hotels and restaurants can be done by the:


oo. Self-administered questionnaires
pp. Office assistant
qq. Manager
rr. None of the given options
21-Discrete variable is also called.
A. Categorical variable
B. Discontinuous variable
C. Both A & B
D. None of the above
22-Officers in my organization have higher than average level of commitment
Such a
hypothesis is an example of.
A. Descriptive Hypothesis
B. Directional Hypothesis
C. Relational Hypothesis
D. All of the above
23-Science refers to.
A. A system for producing knowledge
B. The knowledge produced by a system
C. Both A & B
D. None of the above

24-Which one of the following is not a characteristic of scientific method?


A. Deterministic
Research Method (STA 630) Success Objectives
B. Rationalism
C. Empirical
D. Abstraction
25-The theoretical framework discusses the interrelationships among the.
A. Variables
B. Hypothesis
C. Concept
D. Theory
26-research is based on naturalism.
A. Field research
B. Descriptive research
C. Basic research
D. Applied research
27-Personal interviews conducted in shopping malls are known as
E. Mall interviews
F. Mall intercept interviews
G. Brief interviews
H. None of the given options
28- is used to obtain the freest opinion of the respondent, by asking general
question before a specific question.
I. Research technique
J. Qualitative technique
K. Funnel technique
L. Quantitative technique
29-In, ___________the interviewer and members jointly control the pace and
direction of
the interview.
M. Field interview

N. Telephonic interview
O. Both A and B
P. None of the given options
30-Randomization of test units is a part of
Q. Pretest
R. Posttest
S. Matching
T. Experiment
31- Rationalism is the application of which of the following?
A. Logic and arguments
B. Research solution
C. Reasoning
D. Previous findings
32- On which of the following, scientific knowledge mostly relies?
A. Logical understanding
B. Identification of events
C. Prior knowledge
D. All of the given options
33- Which of the following refers to research supported by measurable
evidence?
A. Opinion
B. Empiricism
C. Speculation
D. Rationalism

34-Research method is applicable in all of the following fields, EXCEPT;


A. Health care
B. Religion
Research Method (STA 630) Success Objectives
C. Business
D. Government offices

35- All of the following are true statements about action research, EXCEPT;
A. Data are systematically analyzed
B. Data are collected systematically
C. Results are generalizable
D. Results are used to improve practice
36-Which of the following is characteristic of action research?
A. Variables are tightly controlled
B. Results are generalizable
C. Data are usually qualitative
D. Results demonstrate cause-and-effect relationships

37-If a researcher is studying the effect of using laptops in his classroom to ascertain
their
merit and worth; he is likely conducting which of the following types of research?
A. Experimental
B. Applied
C. Basic
D. Evaluation
38- Exploratory research addresses which of the following types of question?
A. If
B. How
C. Why
D. What
39- Which of the following is not the source for getting information for exploratory
research?
A. Content analysis
B. Survey

C. Case study
D. Pilot study
40- Which of the following is the main quality of a good theory?
A. A theory that has survived attempts at falsification
B. A theory that is proven to be right
C. A theory that has been disproved
D. A theory that has been falsified
41- A variable that is presumed to cause a change in another variable is known as:
A. Discontinuous variable
B. Dependent variable
C. Independent variable
D. Intervening variable
42- Which of the following is the opposite of a variable?
A. An extraneous variable
B. A dependent variable
C. A data set
D. A constant
43- Which of the following is not a concept?
A. Leadership
B. Total Quality Management
C. Intelligence Quotient (IQ)
D. Human Resource Management
44- Which of the following can best be described as a categorical variable?
A. Age
B. Annual income
C. Grade point average
D. Religion
Research Method (STA 630) Success Objectives
45-Income distribution of employees in a specific organization is an example of
which of

following type of variable?


A. Discontinuous variable
B. Continuous variable
C. Dependent variable
D. Independent variable
46-There is no relationship between higher motivation level and higher efficiency
is an
example of which type of hypothesis?
A. Alternative
B. Null
C. Co relational
D. Research
47- Which of the following is not a role of hypothesis?
A. Guides the direction of the study
B. Determine feasibility of conducting the study
C. Identifies relevant and irrelevant facts
D. Provides framework for organizing the conclusions

48-Hypothesis test may also be called as:


A. Informal test
B. Significance test
C. Moderating test
D. T-test
49-Which type of review compares how different theories address an issue?
A. Context review
B. Integrated review
C. Theoretical review
D. Methodological review

50-After you locate a source, you should write down all details of the reference,
EXCEPT;
A. Volumes
B. Titles
C. Price
D. Full names of the authors
51- ___________research is based on naturalism.
A. Field research
B. Descriptive research
C. Basic research
D. Applied research
52- Personal interviews conducted in shopping malls are known as_________
A. Mall interviews
B. Mall intercepts interviews
C. Brief interviews
D. None of the given options
53- ____________is used to obtain the freest opinion of the respondent, by asking
general question before a specific question.
A. Research technique
B. Qualitative technique
C. Funnel technique
D. Quantitative technique
54- In, ____________the interviewer and members jointly control the pace and
direction of the interview.
A. Field interview
B. Telephonic interview
C. Both A and B
D. None of the given options
Research Method (STA 630) Success Objectives

55- Randomization of test units is a part of ______________


A. Pretest
B. Posttest
C. Matching
D. Experiment
56- Which one of the following sets is the measure of central tendency?
a. Mean, standard deviation, mode
b. Mean, median, standard deviation
c. Arithmetic mean, median, mode
d. Standard deviation, internal validity, mode
57- Internal validity refers to.
a. Researchers degree of confidence.
b. Generalisability
c. Operationalization
d. All of the above
58- How many times the students appear in the research class is the example of
_________.
a. Intensity
b. Space
c. Frequency
d. Direction

59- Time consumed in mall intercept interview is .


a. High
b. Moderate
c. Low
d. Nil

60- Departmental stores selected to test a new merchandising display system is


the example of .
a. Quota sampling
b. Convenience sampling
c. Judgmental sampling
d. Purposive sampling

image: http://api.ning.com/files/Y8bD62HksPhEE7cCSUEuJu-WWaHFBRwNPfyy1UtYNOyNrQktIZ2AMYWpS9eMY97XjvEcA-6xr70r-3y0coSYoP07EKp3Ykim/1075850909.png?xgip=0%3A0%3A189%3A189%3B
%3B&width=48&height=48&crop=1%3A1

image: http://api.ning.com/files/bR8nA-I0SJG6CIZy-G9bFxGXvn7hyT4N02K91fZsBDwSAIVAhU74oiWF5BLjEKrNuuwmyDKushBjV7-opkYm*m2jWzSgMBAS/ddd.png?size=64&crop=1%3A1

Permalink Reply by + M.Tariq Malik on April 1, 2012 at 8:24pm

61- In ___________, the researcher attempts to control and/ or manipulate the


variables in
the study.
1. Experiment
2. Hypothesis
3. Theoretical framework
4. Research design
62- In an experimental research study, the primary goal is to isolate and identify the
effect
produced by the ____.
1. Dependent variable
2. Extraneous variable
3. Independent variable
4. Confounding variable
63- A measure is reliable if it provides consistent ___________.
1. Hypothesis
2. Results
3. Procedure

4. Sensitivity

64- The interview in which questions are already prepared is called ________.
1. Telephonic interview
2. Personal interview
Research Method (STA 630) Success Objectives
3. Unstructured interview
4. Structured interview
65-The numerical description that describe sample may be expected to differ from
those
that describe population because of random fluctuations inherent in sampling
process.
1. Sampling design
2. Non-probability sampling
3. Sampling error
4. Probability sampling
66- In ______________ , each population element has a known and equal chance of
selection.
1. Purposive sampling
2. Quota sampling
3. Stratified sampling
4. Simple random sampling
67- ______ is the evidence that the instrument, techniques, or process used to
measure
concept does indeed measure the intended concepts.
1. Reliability
2. Replicability
3. Scaling
4. Validity

68- A researcher is interested in studying why the new math of the 1960s failed.
She
interviews several teachers who used the new math during the 1960s. These teachers
are
considered as:
1. Primary sources
2. Secondary Sources
3. External critics
4. Internal critics

69- Which of the following is NOT true about stratified random sampling?
1. It involves a random selection process from identified subgroups
2. Proportions of groups in the sample must always match their population
proportions
3. Disproportional stratified random sampling is especially helpful for getting large
enough subgroup samples when subgroup comparisons are to be done
4. Proportional stratified random sampling yields a representative sample
70- Experimental design is the only appropriate design where_________ relationship
can
be established.
1. Strong
2. Linear
3. Weak
4. Cause and Effect
71. Discrete variable is also called.
E. Categorical variable
F. Discontinuous variable
G. Both A & B
H. None of the above
72.Officers in my organization have higher than average level of commitment
Such a hypothesis is an example of.

E. Descriptive Hypothesis
F. Directional Hypothesis
G. Relational Hypothesis
H. All of the above
73.Science refers to.
E. A system for producing knowledge
Research Method (STA 630) Success Objectives
F. The knowledge produced by a system
G. Both A & B
H. None of the above

74.Which one of the following is not a characteristic of scientific method?


E. Deterministic
F. Rationalism
G. Empirical
H. Abstraction
73.The theoretical framework discusses the interrelationships among the.
E. Variables
F. Hypothesis
G. Concept
H. Theory
74.research is based on naturalism.
U. Field research
V. Descriptive research
W. Basic research
X. Applied research
75.Personal interviews conducted in shopping malls are known as
Y. Mall interviews
Z. Mall intercept interviews
AA. Brief interviews

BB. None of the given options


76. is used to obtain the freest opinion of the respondent, by asking general
question before a specific question.
CC. Research technique
DD. Qualitative technique
EE. Funnel technique
FF. Quantitative technique
77.In, the interviewer and members jointly control the pace and direction of
the
interview.
GG. Field interview
HH. Telephonic interview
II. Both A and B
JJ. None of the given options
78.Randomization of test units is a part of
KK. Pretest
LL. Posttest
MM. Matching
NN. Experiment
79. In ___________, the researcher attempts to control and/ or manipulate the
variables in
the study.
1. Experiment
2. Hypothesis
3. Theoretical framework
4. Research design
80. In an experimental research study, the primary goal is to isolate and identify the
effect
produced by the ____.
1. Dependent variable
2. Extraneous variable
3. Independent variable

4. Confounding variable
81. A measure is reliable if it provides consistent ___________.
Research Method (STA 630) Success Objectives
1. Hypothesis
2. Results
3. Procedure
4. Sensitivity
82. The interview in which questions are already prepared is called ________.
1. Telephonic interview
2. Personal interview
3. Unstructured interview
4. Structured interview

83. The numerical description that describe sample may be expected to differ from
those
that describe population because of random fluctuations inherent in sampling
process.
1. Sampling design
2. Non-probability sampling
3. Sampling error
4. Probability sampling
84. In ______________ , each population element has a known and equal chance of
selection.
1. Purposive sampling
2. Quota sampling
3. Stratified sampling
4. Simple random sampling

85. ______ is the evidence that the instrument, techniques, or process used to
measure
concept does indeed measure the intended concepts.
1. Reliability
2. Replicability
3. Scaling
4. Validity
86. A researcher is interested in studying why the new math of the 1960s failed.
She
interviews several teachers who used the new math during the 1960s. These teachers
are
considered as:
1. Primary sources
2. Secondary Sources
3. External critics
4. Internal critics
87. Which of the following is NOT true about stratified random sampling?
1. It involves a random selection process from identified subgroups
2. Proportions of groups in the sample must always match their population
proportions
3. Disproportional stratified random sampling is especially helpful for getting large
enough subgroup samples when subgroup comparisons are to be done
4. Proportional stratified random sampling yields a representative sample
88. Experimental design is the only appropriate design where_________ relationship
can
be established.
1. Strong
2. Linear
3. Weak
4. Cause and Effect
89. Rationalism is the application of which of the following?
A. Logic and arguments

B. Research solution
C. Reasoning
D. Previous findings
90. On which of the following, scientific knowledge mostly relies?
A. Logical understanding
B. Identification of events
Research Method (STA 630) Success Objectives
C. Prior knowledge
D. All of the given options

91. Which of the following refers to research supported by measurable


evidence?
A. Opinion
B. Empiricism
C. Speculation
D. Rationalism
92. Research method is applicable in all of the following fields, EXCEPT;
A. Health care
B. Religion
C. Business
D. Government offices

93. All of the following are true statements about action research, EXCEPT;
A. Data are systematically analyzed
B. Data are collected systematically
C. Results are generalizable
D. Results are used to improve practice
94. Which of the following is characteristic of action research?

A. Variables are tightly controlled


B. Results are generalizable
C. Data are usually qualitative
D. Results demonstrate cause-and-effect relationship
95. If a researcher is studying the effect of using laptops in his classroom to
ascertain their merit and worth, he is likely conducting which of the
following types of research?
A. Experimental
B. Applied
C. Basic
D. Evaluation
96. Exploratory research addresses which of the following types of
question?
A. If
B. How
C. Why
D. What
97. Which of the following is not the source for getting information for
exploratory research?
A. Content analysis
B. Survey
C. Case study
D. Pilot study

98. Which of the following is the main quality of a good theory?


A. A theory that has survived attempts at falsification
B. A theory that is proven to be right
C. A theory that has been disproved
D. A theory that has been falsified

100. Which of the following is not a concept?


A. Leadership
B. Total Quality Management
C. Intelligence Quotient (IQ)
D. Human Resource Management
101. A variable that is presumed to cause a change in another variable is
known as:
Research Method (STA 630) Success Objectives
A. Discontinuous variable
B. Dependent variable
C. Independent variable
D. Intervening variable
102. Which of the following is the opposite of a variable?
A. An extraneous variable
B. A dependent variable
C. A data set
D. A constant
103. Which of the following can best be described as a categorical
variable?
A. Age
B. Annual income
C. Grade point average
D. Religion

104. Income distribution of employees in a specific organization is an


example of which of following type of variable?
A. Discontinuous variable
B. Continuous variable
C. Dependent variable

D. Independent variable
http://.com
105. There is no relationship between higher motivation level and higher
efficiency is an example of which type of hypothesis?
A. Alternative
B. Null
C. Correlational
D. Research
106. Which of the following is not a role of hypothesis?
A. Guides the direction of the study
B. Determine feasibility of conducting the study
C. Identifies relevant and irrelevant facts
D. Provides framework for organizing the conclusions
107. Which type of review compares how different theories address an issue?
A. Context review
B. Integrated review
C. Theoretical review
D. Methodological review
108. After you locate a source, you should write down all details of the
reference, EXCEPT;
A. Volumes
B. Titles
C. Price
D. Full names of the authors

109. What is the primary focus of establishment surveys in this case study?
A. Collect the data through past studies
B. Analyze the literature review

C. Using of quantitative techniques


D. Data collection through mail and Interview
110. Which one of following is generally common in establishment survey and
household
survey?
A. Cognitive recall
B. Homogenous respondents
C. Error free
D. Response burden
Research Method (STA 630) Success Objectives
111. Which one of the following is not of important consideration in establishment
survey
while designing questionnaires?
A. Response burden
B. Professional terminology
C. Cognitive recall
D. Use of Records
112. Which of the following method of data collection is not discussed in the case
study?
A. Questionnaires
B. Interviews
C. Mail survey
D. Observations
113. Which of the following sampling technique is used for Employee Turnover and
Job
Openings survey?
A. Simple random sampling
B. Cluster sampling
C. Stratified sampling
D. Convenience sampling

114. Which one of the following is the limitation of establishment survey in this case
study?
A. Cost
B. Limited data
C. Unskilled interviewer
D. Small sample size
http://.com
115. Which of the following is not the part of specific protocol of focus groups in
ETJO?
A. Concept and indicators
B. Definition
C. Availability of records
D. Cognitive recall
116. Which of the following is the draw back of pretest interview in ETJO survey?
A. Small simple size
B. Non cooperative response
C. Probing
D. Questionnaire format
117. Which of the following method of data collection is not used in the case study?
A. Questionnaires
B. Focus groups
C. Correlational method
D. Secondary data
118. What is the basic purpose of ETJO survey?
A. To assess the feasibility of collecting job-vacancy and turnover data by
occupation
B. To analyze the problem of labor shortage
C. To assess the motivation level of employees
D. To analyze the factor contributing towards employee turnover

119. Which of the following is the basic purpose of pretest interview in this case
study?
A. To identified the potential problem
B. To know the sample size
C. To develop the questionnaire
D. To use agency representative
120. Which one of the following sampling type is used in operations test to select the
units?
A. Simple random sampling
Research Method (STA 630) Success Objectives
B. Cluster sampling
C. Quota sampling
D. Judgment sampling

image: http://api.ning.com/files/Y8bD62HksPhEE7cCSUEuJu-WWaHFBRwNPfyy1UtYNOyNrQktIZ2AMYWpS9eMY97XjvEcA-6xr70r-3y0coSYoP07EKp3Ykim/1075850909.png?xgip=0%3A0%3A189%3A189%3B
%3B&width=48&height=48&crop=1%3A1

image: http://api.ning.com/files/bR8nA-I0SJG6CIZy-G9bFxGXvn7hyT4N02K91fZsBDwSAIVAhU74oiWF5BLjEKrNuuwmyDKushBjV7-opkYm*m2jWzSgMBAS/ddd.png?size=64&crop=1%3A1

Permalink Reply by + M.Tariq Malik on April 1, 2012 at 8:27pm

121. Which of the following is the basic purpose of Response analysis survey in the
case
study?
A. To assess the quality of ETJO survey data
B. To know the sample size of ETJO survey data
C. To develop the questionnaire for ETJO
D. To use agency representative for ETJO
122. After Operation test, which of the following test findings were suggested by the
researcher?
A. Need of highly skilled and well trained interviewer

B. Sample size should be increased


C. A decent increase in survey budget
D. Focus group should be included
123. In which one of the following stage researcher consult the literature?
A. Operation test
B. Response analysis survey
C. Document design analysis
D. Pretest interviews

124. Which one of the following sampling type is used in Response analysis survey
(RAS)?
A. Simple random sampling
B. Cluster sampling
C. Quota sampling
D. Stratified sampling
125. Which one of the following could be helpful for minimizing the bias in this case
study?
A. Cognitive research
B. Focus group
C. Pretest Interview
D. Response analysis survey
126. Which one of the following is useful in assessing and clarifying concepts and
definitions at the beginning stages of questionnaire?
A. Operation test
B. Document design analysis
C. Focus group
D. Response analysis survey

127. Which one of the following can be more helpful than others in order to
determine the
exact source of measurement errors in establishment survey?
A. Focus group
B. Operation test
C. Response analysis survey
D. Document design analysis
Identify the True and False statements. (05)
(1) Review of literature shows the incredibility of the body of knowledge and identifies
the
points at which the researcher made the research area ambiguous and unclear. (False)
(2) The literature survey provides a solid foundation for developing the theoretical
framework. (True)
(3) The hypothesis should not only be specific to a place and situation but also these
should
be narrowed down with respect to its operation. (True )
(4) A cohort analysis is similar to cross-sectional research. (False)
Research Method (STA 630) Success Objectives
(5) A literature review does not consider the prior studies, their agreements or
disagreements in the field. (False)
Fill in the blanks with appropriate words.
(1) Research hypothesis is formulated by the researcher which suggests the nature of
relationship i.e. the direction of relationship.
(2) Correlational hypothesis merely states that the variables occur together and does not
imply that one causes the other.
(3) Longitudinal type of research examine feature of any unit at more than one time.
(4) Historical review traces the development of an idea or shows how a particular issue
or
theory has evolved over time.
(5) Discontinuous variable is also known as discrete/ categorical/ classificatoryvariable.
6. Validity problems occur when the researchers theoretical definition does not
match that of the government agency or organization that collected the
information.
7. Reliability problems occur when official definition or the method of collecting

Information changes over time.


8. Secondary data refer to information gathered by someone other than the
researcher conducting the present study.
9. Experiments are best for topics where the researcher controls a situation and
manipulates an Independent variable.
10. A social indicator is any measure of wellbeing used in policy.
11.Two power bases of scientific knowledge are Empiricism and Rationalism
12.Action research is a type of Applied research
13.Three types of longitudinal research are time series, panel, and cohort
14.The research proposal is just like a research report, but it is written before the research
project begins.
15.Natural science concepts are often expressed in symbolic forms.
16.The double blind experiment is designed to control experimenter expectancy.
17.Pilot testing is also called pre-testing.
18.The totality, out of which sample is drawn is referred to as population.
19.The physical setting of the interview creates Situational bias.
20.Probing is the verbal prompts made by field worker.

http://.com
Long Questions:
Q1. Write any five similarities of Historical-Comparative research and Field
research?
Answer:
1. Both H-C research and field research recognize that the researchers point of view
is an avoidable part of research. Both involve interpretation, which introduce the
interpreters location in time, place, and world-view.
2. Both field and H-C research examine a great diversity of data. In both the
research becomes immersed in data to gain an emphatic understanding of events
and people.
3. Both field and H-C research often use grounded theory (theory usually emerges
during the process of data collection.)
4. Both field and H-C research involves a type of translation. The researchers
meaning system usually differs from that of people he or she studies, but he or she

Research Method (STA 630) Success Objectives


tries to penetrate and understand their point of view.
5. Both field and H-C research focus on action, process, and sequence and see time
process as essential.
6. Generalization and theory are limited in field and H-C research.
Q2. How quantitative and qualitative techniques are applied in HistoricalComparative research.
Answer:
Quantitative approach
The quantitative techniques with some minor adjustment can be applied to study the past
or other cultures.
_ The researcher can focus on the issue in one society, few societies or multiple
societies.
_ The researcher can focus on the issue in one time in the past or examine the issue
across many years/periods in the past.
_ The researcher can focus on the issue in the present or a recent past period.
_ The researchers analysis could be based primarily on quantitative data or
qualitative data.
Qualitative approach
_ The researcher must be able to take on, mentally, the circumstances, views, and
feelings of those being studied to interpret their action appropriately.
_ The historical-comparative research must find patterns among the voluminous
details describing the subject matter of study.
Question # 01
How do concrete and abstract concepts differ? Give example.
(Marks: 04)
Solution:
Concrete Concepts:
Concrete ones refer to straightforward physical objects or familiar experiences.
Abstract concepts:
Abstract concepts refer to ideas that have a scattered meaning or an indirect expression
Abstraction is a thought process wherein ideas are distanced from objects.

The difference between concrete and abstract concepts can be expressed with some
characterizations like,
1. Abstract concepts have only relational properties while concrete concepts have some
fundamental properties.
2. Abstract concepts are universals and concrete concepts are particulars.
3. Abstract concepts are sets (generalized) and concrete concepts are individuals
(specified).
4. Concrete concepts are known by observation whereas abstract concepts are known in
some other way, e.g., by abstraction, by intuition, etc
Example
Print Media (Abstract)
Publication
News Paper
Daily Dawn (Concrete)
Question # 02
Research Method (STA 630) Success Objectives
Given the situation below, Discuss with reason into which category they fall
According to the purpose of doing research and
The uses of research
(Marks: 03, 03)
Scenario 1:
Reasons for Absenteeism
A university professor wanted to analyze in depth the reasons for absenteeism of
employees
in organizations. Fortunately, a company within 20 miles of the campus employed her as
a
consultant to study that very issue.
Solution:
According to the purpose of doing research :
It is Explanatory research as the purpose is to find out the reasons for a phenomenon
i.e.

absenteeism of employees in organizations. Research seeks to determine the answers to


why question about a specific behavior.
According to the uses of research:

It is applied research since, the problem is being faced by an organization and the
research
is called upon to solve it.
Scenario 2:
Effects of Nasal Spray on Flu
A research scientist surveys 1,000 employees in different organizational settings to study
the
efficacy of several types of nasal sprays in controlling the flu virus. He subsequently
publishes his findings in a highly respected medical journal.
Solution:
According to the purpose of doing research :
It is Descriptive research, as, the research question only finds out the special effects of

several types of nasal sprays in controlling the flu virus in different organizational
settings. It
is to find out the relationship among the substances under study.
According to the uses of research:
It is a Basic research since the underlying purpose is to expand the frontier of knowledge
.
Question#01:
Why turn over is high in some organizations?
As per above given query, explain the following components of Theoretical frame
work;
(Marks: 10)
i) Develop an inventory of variables (min. five variables required)
ii) Specify direction of relationship
iii) Give clear explanation why you should expect the proposed relationship exists
iv) Make an inventory of propositions
Ans:
i) Develop an inventory of variables
Salary Package is low, Inflexibility in working hours, Low decentralization (participation
in
decision making), Lack of other benefits (bonus, appraisals, conveyance facilities, etc),
Poor
working ambiance
ii) Specify direction of relationship
a) Between salary & turn-over negative relationship exists

b) Between flexible working hours & turn-over negative relationship exists


c) Between decentralization & turn-over negative relationship exists
Research Method (STA 630) Success Objectives
d) Between extra benefits & turn-over negative relationship exists
e) Between ambiance & turn-over negative relationship exists
iii) Give clear explanation why you should expect the proposed relationship exists?
a) Between salary & turn-over negative relationship exists because in current era a good
salary package has more attraction for employees to remain in the same organization. So
organizations that keep themselves up-dated with privileged market salary packages
usually have low turn over ratio.
b) Between flexible working hours & turn-over negative relationship exists because
flexibility
in working hours give some relaxation to employees especially in case when their targets
are not achieved due to a reason able fact.
c) Between decentralization & turn-over negative relationship exists because
participation in
decision making raise confidence of employee and motivation level. So, organization
have more decentralized system face low turn-over ratio.
d) Between extra benefits & turn-over negative relationship exists. The organizations
offer
different packages like bonuses, conveyance facilities etc, such organizations have more
satisfies employees and low turn over.
e) Between ambiance & turn-over negative relationship exists. Working ambiance does
matter a lot. Good ambiance that help in releasing working stress increase the employee
motivation and interest in doing job well.

image: http://api.ning.com/files/Y8bD62HksPhEE7cCSUEuJu-WWaHFBRwNPfyy1UtYNOyNrQktIZ2AMYWpS9eMY97XjvEcA-6xr70r-3y0coSYoP07EKp3Ykim/1075850909.png?xgip=0%3A0%3A189%3A189%3B
%3B&width=48&height=48&crop=1%3A1

image: http://api.ning.com/files/bR8nA-I0SJG6CIZy-G9bFxGXvn7hyT4N02K91fZsBDwSAIVAhU74oiWF5BLjEKrNuuwmyDKushBjV7-opkYm*m2jWzSgMBAS/ddd.png?size=64&crop=1%3A1

Permalink Reply by + M.Tariq Malik on April 1, 2012 at 8:28pm

Question#02:
What is meant by a Research Proposal? Explain briefly. Also mention at what time a
research proposal is submitted? (Marks: 05)
Note: (Copied data from hand outs will not be appreciated)
Ans:
A research proposal is a document that plans how and in which manner a research will be
conducted. Research proposals contain extensive literature reviews. Such proposals must
offer
convincing support for your research topic. It means the topic/issue for which research
study is
being proposed should be sound enough to conduct a worthy research. The proposal must
describe a detailed methodology for conducting the research i.e. which tools are going to
be used
in your research.
More over when writing a proposal it is well to assume that reader of your proposal is
interested
to find out the answer of these questions;

What do you want to do, how much will it cost, and how much time will it take?
What has already been done in the area of your project?
How do you plan to do it?
How will the results be evaluated?

A good proposal is that which well defines these questions and convinces the reader that
research on specified issue is needed to conduct. So, the research proposal is submitted
before
starting the research project because in this you plan how the data will be collected and
analyzed.
Q. Explain the difference between these terms with one example of each.
i. Population and target population
ii. Sampling unit and observation unit
iii. Parameter and statistic
i. Population and Target Population.
Population refers to the entire group of people, events or things of interest that the

researcher wishes to investigate.


Target population is the complete group of specific population elements relevant to the
research project.
Example
Suppose a researcher wants to investigate the productivity of employees working in
multinationals in Pakistan.
All the employees working in multinational will be the population.
The researcher wants to investigate the productivity of employees working in
multinational in FMCG sector in Pakistan.
Research Method (STA 630) Success Objectives
All the employees working in multinationals in FMCG sector in Pakistan will be the
target population.
ii. Sampling Unit and Observation Unit
A sampling unit is that element or set of elements considered for selection in some stage
of sampling.
An observation unit is an element or aggregation of elements from which the
information is collected.
Example
In the above example the researcher selects employees from three FMCG multinationals
(Procter and Gamble, Unilever and Nestle) for data collection. Among these employees
he selected First Line Managers for collecting the data. Among all First Line Manager he
selects 40 Line managers for data collection.
In this example the employees from three FMCGs (Procter and Gamble, Unilever and
Nestle) are the primary sampling unit, all the First Line managers are the secondary/ final
sampling unit and the selected First Line managers are the observation unit.
iii. Parameter and Statistics
A parameter is the summary description of a given variable in a population while
Statistic is the summary description of a given variable in a survey sample.
Example
In the above example the mean productivity of all the employees working in the
multinational is the Parameter and the mean productivity of the First Line managers from
those the researcher collected the data is the Statistics.
Q: Define Research? Discuss the special features of Scientific method of Research?

Solution:
Definition of Research:
Research is an organized and systematic way of finding answers to questions by
gathering the
needed information. General image of the research is that it has something to do with the
laboratory where scientists are supposedly doing some experiments. Research is simply
the
process of finding solutions to a problem after thorough study and analysis of the
situational
factors. In research, we try to follow a system or a procedure in an organized manner. It is
all the
more necessary in case we want to repeat the study, or somebody else wants to verify our
findings. In the latter case the other person has to follow the same procedure that we
followed.
Hence not only we have to do the study in a systematic manner but also that system
should be
known to others.
Special Features of Scientific Method of Research:
The Scientific method of Research is a procedure to produce knowledge and to discover
the
principles & laws in this universe.
Special Features of Scientific Method of Research are as follows:
1. Empirical
Scientific method is concerned with the realities that are observable through sensory
experiences. It generates knowledge which is verifiable by experience or observation.
Some of
the realities could be directly observed, like the number of students present in the class
and how
many of them are male and how many female. The same students have attitudes, values,
motivations, aspirations, and commitments. These are also realities which cannot be
observed
directly, but the researchers have designed ways to observe these indirectly. Any reality
that
cannot be put to sensory experience directly or indirectly (existence of heaven, the Day
of

Judgment, life hereafter, Gods rewards for good deeds) does not fall within the domain
of
scientific method.
2. Verifiable
Observations made through scientific method are to be verified again by using the senses
to
confirm or refute the previous findings. Such confirmations may have to be made by the
same
researcher or others. We will place more faith and credence in those findings and
conclusions if
similar findings emerge on the basis of data collected by other researchers using the same
methods. To the extent that it does happen (i.e. the results are replicated or repeated) we
will
gain confidence in the scientific nature of our research. Replicability, in this way, is an
important
characteristic of scientific method. Hence revelations and intuitions are out of the domain
of
scientific method.
3. Cumulative
Prior to the start of any study the researchers try to scan through the literature and see that
their
study is not a repetition in ignorance. Instead of reinventing the wheel the researchers
take stock
of the existing body of knowledge and try to build on it. Also the researchers do
Research Method (STA 630) Success Objectives
not leave their research findings into scattered bits and pieces. Facts and figures are to be
provided with language and thereby inferences drawn. The results are to be organized and
systematized. Nevertheless, we dont want to leave our studies as stand alone. A linkage
between the present and the previous body of knowledge has to be established, and that is
how
the knowledge accumulates.
4. Deterministic
Science is based on the assumption that all events have antecedent causes that are subject
to
identification and logical understanding. For the scientist, nothing just happens it
happens for

a reason. The scientific researchers try to explain the emerging phenomenon by


identifying its
causes. Of the identified causes which ones can be the most important? For example, in
the
2006 BA/BS examination of the Punjab University 67 percent of the students failed.
What could
be the determinants of such a mass failure of students? The researcher may try to explain
this
phenomenon and come up with variety of reasons which may pertain to students,
teachers,
administration, curriculum, books, examination system, and so on. Looking into such a
large
number of reasons may be highly cumbersome model for problem solution. It might be
appropriate to tell, of all these factors which one is the most important. The researcher
tries to
narrow down the number of reasons in such a way that some action could taken.
5. Ethical and Ideological Neutrality
The conclusions drawn through interpretation of the results of data analysis should be
objective;
that is, they should be based on the facts of the findings derived from actual data, and not
on our
own subjective or emotional values. For instance, if we had a hypothesis that stated that
greater
participation in decision making will increase organizational commitment, and this was
not
supported by the results, it makes no sense if the researcher continues to argue that
increased
opportunities for employee participation would still help. Such an argument would be
based, not
on the factual, data based research findings, but on the subjective opinion of the
researcher. If
this was the conviction of the researcher all along, then there was no need to do the
research in
the first place. Researchers are human beings, having individual ideologies, religious
affiliations,

cultural differences which can influence the research findings. Any interference of their
personal
likings and dis-likings in their research can contaminate the purity of the data, which
ultimately
can affect the predictions made by the researcher. Therefore, one of the important
characteristics of scientific method is to follow the principle of objectivity, uphold
neutrality, and
present the results in an unbiased manner.

6. Statistical Generalization
Generalisability refers to the scope of the research findings in one organizational setting
to other
settings. Obviously, the wider the range of applicability of the solutions generated by
research,
the more useful the research is to users. For instance, if a researchers findings that
participation
in decision making enhances organizational commitment are found to be true in a variety
of
manufacturing, industrial, and service organizations, and not merely in the particular
organization
studied by the researcher, the generalisability of the findings to other organizational
settings is
enhanced. The more generalizable the research, the greater its usefulness and value.
For wider generalisability, the research sampling design has to be logically developed.
Here the
use of statistics is very helpful. Statistics is device for comparing what is observed and
what is
logically expected. The use of statistics becomes helpful in making generalizations,
which is one
of the goals of scientific method.
7. Rationalism
Science is fundamentally a rational activity, and the scientific explanation must make
sense.
Religion may rest on revelations, custom, or traditions, gambling on faith, but science
must rest

on logical reason. There are two distinct logical systems important to the scientific quest,
referred
to as deductive logic and inductive logic.
Q: Define Hypothesis? What are different types of hypothesis? Explain.
Solution:
Definition of Hypothesis:
A proposition that can be verified to determine its reality is a hypothesis. A hypothesis
may be
defined as a logically conjectured relationship between two or more variables, expressed
in the
form of a testable statement. Relationship is proposed by using a strong logical
argumentation.
This logical relationship may be part of theoretical framework of the study.
For example,

Officers in my organization have higher than average level of commitment


(variable).
Level of job commitment of the officers is associated with their level of
efficiency.
Level of job commitment of the officers is positively associated with their level of

efficiency.
Research Method (STA 630) Success Objectives

The higher the level of job commitment of the officers the lower their level of

absenteeism.
Types of Hypothesis:
There are different types of hypothesis which are as follows.
1-Descriptive Hypothesis
Descriptive hypothesis contains only one variable thereby it is also called as univariate
hypothesis. Descriptive hypotheses typically state the existence, size, form, or
distribution of
some variable. The first hypothesis contains only one variable. It only shows the
distribution of
the level of commitment among the officers of the organization which is higher than
average.
Such a hypothesis is an example of a Descriptive Hypothesis.
2- Relational Hypothesis

These are the propositions that describe a relationship between two variables. The
relationship
could be non-directional or directional, positive or negative, causal or simply
correlational.
While stating the relationship between the two variables, if the terms of positive,
negative, more
than, or less than are used then such hypotheses are directional because the direction of
the
relationship between the variables (positive/negative) has been indicated. These
hypotheses are
relational as well as directional. The directional hypothesis is the one in which the
direction of the
relationship has been specified.
Non-directional hypothesis is the one in which the direction of the association has not
been
specified. The relationship may be very strong but whether it is positive or negative has
not been
postulated.
Correlational hypotheses state merely that the variables occur together in some
specified
manner without implying that one causes the other. Such weak claims are often made
when we
believe that there are more basic causal forces that affect both variables. For example:
Level of job commitment of the officers is positively associated with their level of
efficiency.
Explanatory (causal) hypotheses imply the existence of, or a change in, one variable
causes or
leads to a change in the other variable. This brings in the notions of independent and the
dependent variables. Cause means to help make happen. So the independent variable
may
not be the sole reason for the existence of, or change in the dependent variable.
3- Null Hypothesis
It is used for testing the hypothesis formulated by the researcher. Researchers treat
evidence that

supports a hypothesis differently from the evidence that opposes it. They give negative
evidence
more importance than to the positive one. It is because the negative evidence tarnishes
the
hypothesis. It shows that the predictions made by the hypothesis are wrong. The null
hypothesis
simply states that there is no relationship between the variables or the relationship
between the
variables is zero. That is how symbolically null hypothesis is denoted as H0. For
example:
H0 = There is no relationship between the level of job commitment and the level of
efficiency. Or
H0 = The relationship between level of job commitment and the level of efficiency is
zero. Or the
two variables are independent of each other.
It does not take into consideration the direction of association (i.e. H0 is non directional),
which
may be a second step in testing the hypothesis.
4- Alternative Hypothesis
The alternative (to the null) hypothesis simply states that there is a relationship between
the
variables under study. In our example it could be: there is a relationship between the level
of job
commitment and the level of efficiency. Not only there is an association between the two
variables under study but also the relationship is perfect which is indicated by the number
1.
Thereby the alternative hypothesis is symbolically denoted as H1. It can be written like
this:
H1: There is a relationship between the level of job commitment of the officers and their
level of
efficiency.
5- Research Hypothesis
Research hypothesis is the actual hypothesis formulated by the researcher which may also
suggest the nature of relationship i.e. the direction of relationship. In our example it could
be:

Level of job commitment of the officers is positively associated with their level of
efficiency.
Q: Ahmad is the manager of a restaurant; he wants to see his staff cooperative and
hardworking but his staff is not working well according to his will. He decided to
give the
incentives in order to motivate and encourage them but his policy did not work
effectively.
There can be many factors behind this behavior of staff that needs to be researched.
For
this purpose, he appoints you as a researcher and asks you to discover those factors
and
submit a report to him. How you will undertake this kind of
Research Method (STA 630) Success Objectives
research and what steps you will choose to make a report about the behavior of the
staff?
Solution:
The manager of the restaurant appoints me as a researcher to find out the factors behind
the staff
behavior. I will take the following steps in order to undertake this research and to make a
report.
These steps are as follows:
STEP 1: To Select the topic of research:
First of all I will select the topic which is a general area of study or issue. The topic in
this
research is the factors behind the staff behavior. A topic appears to be too broad for
conducting
research. The specific issues that need to be researched within the situation may not be
identified at this stage. I will try to analyze the problems currently existing in the
restaurant that
needs to be solved and the areas that a manager believes need to be improved in the
restaurant
(improving the existing policies). Then I will develop some research questions that a
basic
researcher wants to answer empirically.
STEP 2: Preliminary Data Collection or to Explore different areas from where I can
get
information related to the restaurant and staff behavior:

After selecting the topic, I will collect data from different sources. This step may be
considered as
part of the exploratory research. An exploration typically begins with a search for
published data
and studies. Such sources can provide secondary data which becomes part of the
background
information (about the restaurant & groups of people). Some secondary sources of data
are
statistical bulletins, government publications, information published or unpublished, case
studies,
online data, web sites, and the Internet from where I can get information. In addition,
information
can also be collected from the people who are well informed about the staff and
restaurant,
especially those who have clearly stated positions on controversial aspects of the
problem. In
certain situations, I will also focus on group discussions with the staff of the restaurant.
Such
discussions can help me in the identification of factors and having clarification of their
behavior.
STEP 3: Problem Definition
After having discussions with the professionals as well as with the staff to whom the
issue relates,
and the review of literature, now I will be in a position to narrow down from its original
broad base
and define the factors clearly. Translate the broad issue into a research question. As part
of the
applied research convert the management dilemma into a management question, and then
on to
research question that fits the need to resolve the dilemma. The symptoms of a problem
might
help tracing the real problem. For example in this research, the productivity decline of
workers is
an issue. The manager has tried to solve it by the provision of incentives but his policy
did not

work. I will find the possible factors like the morale and motivation of the workers
having some
other antecedents.
STEP 4: To develop the Theoretical Framework
Consultations with the informants and professionals, and the review of literature will help
me in
the identification of different factors that are considered to be relevant. Now I will need
to make a
logical relationship among several factors identified. This will help in the delineation of
the
theoretical framework. The theoretical framework discusses the interrelationships among
the
variables that are deemed to be integral to the dynamics of the situation being
investigated.
Developing such a conceptual framework helps to postulate or hypothesize and test
certain
relationships.
STEP 5: To Generate the Hypotheses
Once I have identified the important variables relevant to an issue and established the
logical
reasoning in the theoretical framework, I will be in a position to test whether the
relationships that
have been theorized do in fact hold true. By testing these relationships scientifically, I
will obtain
reliable information to determine the relationship among the variables. The results of
these tests
offer part of the answers to the formulated research questions, whether these relate basic
research or to applied research.
STEP 6: To make a Research Design
Research design is a master plan specifying the methods and procedures for collecting
and
analyzing the needed information. It is a framework or the blueprint that plans the action
for
research project. The objectives of the study determined during the early stages of the
research

are included in the design to ensure that the information collected is appropriate for
solving the
problem. It is very important to specify the sources of information, and the research
method or
technique (survey or experiment, for example) to be followed in the study. Broadly there
are six
basic research methods for descriptive and causal research: surveys, experiments,
observation,
communication analysis (content analysis), case study, focus group discussion.
STEP 7: Data Collection, Data Processing, and Analysis
Research Method (STA 630) Success Objectives
Data collection is integral part of the research design; Data collection is determined by
the
research technique selected for the project. Data can be collected in a variety of ways, in
different settings field or lab and from different sources. It could includeinterviews
face to
face interviews, telephone interviews, computer-assisted interviews, and interviews
through
electronic media; questionnaires that either personally administered, sent through mail, or
electronically administered; observation of individuals and events which could be
participant or
non participant. Once the fieldwork has been completed, the data must be converted into
a format
that will answer the research questions and help testing the hypotheses. The computer can
help
in making tables and the application of different statistics.
STEP 8: Testing the Hypotheses; Answering the Research Questions
The analysis and interpretation of the data shall be the means to testing the formulated
hypotheses as well as finding answers to the research questions. In case of applied
research,
the research should be helpful in finding solutions to the problems of the organization or
society.
Making recommendations may also be part of this process.
STEP 9: Report Writing

The research report should communicate the research findings effectively. All too often
the report
is a complicated statement of the studys technical aspects and sophisticated research
methods.
The report has to be presented in the format as it may have been part of the terms of
reference if
it is a sponsored study. I will write the basic factors behind the staff behavior in my report
and I
will also give some suggestions to the manager to motivate the staff so that they can work
efficiently.
stion.<l,<ow(>
A. Research technique
B. Qualitative technique
C. Funnel technique
D. Quantitative technique
54- In, ____________the interviewer and members jointly control the pace and
direction of the interview.
A. Field interview
B. Telephonic interview
C. Both A and B
D. None of the given options
Research Method (STA 630) Success Objectives
55- Randomization of test units is a part of ______________
A. Pretest
B. Posttest
C. Matching
D. Experiment
56- Which one of the following sets is the measure of central tendency?
a. Mean, standard deviation, mode
b. Mean, median, standard deviation
c. Arithmetic mean, median, mode

d. Standard deviation, internal validity, mode


57- Internal validity refers to.
a. Researchers degree of confidence.
b. Generalisability
c. Operationalization
d. All of the above
58- How many times the students appear in the research class is the example of
_________.
a. Intensity
b. Space
c. Frequency
d. Direction

59- Time consumed in mall intercept interview is .


a. High
b. Moderate
c. Low
d. Nil
60- Departmental stores selected to test a new merchandising display system is
the example of .
a. Quota sampling
b. Convenience sampling
c. Judgmental sampling
d. Purposive sampling
Q: Define the measure that is used by the researcher in order to capture the
intensity,
direction, level, or potency of a variable construct. Also describes the different types
of

that measure.
Solution:
The measure that is used by the researcher in order to capture the intensity,
direction,
level, or potency of a variable construct:
A scale is a measure in which a researcher captures the intensity, direction, level, or
potency of a
variable construct. It arranges responses or observations on a continuum or in series of
categories. A scale can use a single indicator or multiple indicators.
An index is a measure in which a researcher adds or combines several distinct indicators
of a
construct into a single score. The composite scores is often a simple sum of the multiple
indicators. Indexes are often measured at the interval or ratio level.
Researchers sometimes combine the features of scales and indexes in a single measure.
This is
common when a researcher has a several indicators that are scales (i.e. that measure
intensity or
direction). The researcher then adds these indicators together to yield a single score,
thereby
creating an index.
The different types of that measure:
A scale refers to any series of items that are arranged progressively according to value or
magnitude, into which an item can be placed according to its quantification. In other
words, a
scale is a continuous spectrum or series of categories.
It is traditional to classify scales of measurement on the basis of the mathematical
comparisons
that are allowable with these scales. Four types of scales are nominal, ordinal, interval,
and ratio.
Nominal Scale
A nominal scale is the one in which the numbers or letters assigned to objects serve as
labels for
identification or classification. This measurement scale is the simplest type. With nominal
data,
we are collecting information on a variable that naturally or by design can be grouped
into two or

more categories that are mutually exclusive, and collectively exhaustive.


Nominal scales are the least powerful of the four scales. They suggest no order or
distance
relationship and have no arithmetic origin. Nevertheless, if no other scale can be used,
one can
almost always one set of properties into a set of equivalent classes.
Ordinal Scale
Research Method (STA 630) Success Objectives
Ordinal scales include the characteristics of the nominal scale plus an indicator of order.
If a is
greater than b and b is greater than c, then a is greater than c. The use of ordinal scale
implies a
statement of greater than or less than without stating how much greater or less. Other
descriptors can be: superior to, happier than, poorer than, or above.
Interval Scale
Interval scales have the power of nominal and ordinal scales plus one additional strength:
they
incorporate the concept of equality of interval (the distance between 1 and 2 equals the
distance
between 2 and 3). For example, the elapsed time between 3 and 6 A. M. equals the time
between 4 and 7 A. M. One cannot say, however, 6 A.M. is twice as late as 3 A.M.
because
zero time is an arbitrary origin. In the consumer price index, if the base year is 1983,
the price
level during 1983 will be set arbitrarily as 100. Although this is an equal interval
measurement
scale, the zero point is arbitrary.
Ratio Scale
Ratio scales incorporate all the powers of the previous scales plus the provision for
absolute zero
or origin. Ratio data represent the actual amounts of variable. Measures of physical
dimensions
such as weight, height, distance, and area are the examples. The absolute zero represents
a

point on the scale where there is an absence of the given attribute. If we hear that a
person has
zero amount of money, we understand the zero value of the amount.
Q: (a) Define the Probability & non-Probability sampling.
Solution (a):
Probability Sampling:
In probability sampling, every element in the population has a known nonzero probability
of
selection. The simple random is the best known probability sample, in which each
member of the
population has an equal probability of being selected.
Non-Probability Sampling:
In non-probability sampling the probability of any particular element of the population
being
chosen is unknown. The selection of units in non-probability sampling is quite arbitrary,
as
researchers rely heavily on personal judgment.
(b) Snowball sampling is also known as network, chain referral or reputational
sampling.
Comment on this statement.
Solution (b):
Snowball sampling:
Snowball sampling is also called network, chain referral, or reputational sampling. It is a
method
for identifying and sampling cases in the network. It is based on an analogy to a
snowball, which
begins small but becomes larger as it is rolled on wet snow and picks up additional snow.
It
begins with one or a few people or cases and spreads out on the basis of links to the
initial cases.
This design has been found quite useful where respondents are difficult to identify and
are best
located through referral networks. In the initial stage of snowball sampling, individuals
are
discovered and may or may not be selected through probability methods. This group is
then used
to locate others who possess similar characteristics and who, in turn, identify others. The

snowball gather subjects as it rolls along.


Example:
For example, a researcher examines friendship networks among teenagers in a
community. He
or she begins with three teenagers who do not know each other. Each teen names four
close
friends. The researcher then goes to the four friends and asks each to name four close
friends,
then goes to those four and does the same thing again, and so forth. Before long, a large
number of people are involved. Each person in the sample is directly or indirectly tied to
the
original teenagers, and several people may have named the same person. The researcher
eventually stops, either because no new names are given, indicating a closed network, or
because the network is so large that it is at thee limit of what he or she can study.

image: http://api.ning.com/files/Y8bD62HksPhEE7cCSUEuJu-WWaHFBRwNPfyy1UtYNOyNrQktIZ2AMYWpS9eMY97XjvEcA-6xr70r-3y0coSYoP07EKp3Ykim/1075850909.png?xgip=0%3A0%3A189%3A189%3B
%3B&width=48&height=48&crop=1%3A1

image: http://api.ning.com/files/bR8nA-I0SJG6CIZy-G9bFxGXvn7hyT4N02K91fZsBDwSAIVAhU74oiWF5BLjEKrNuuwmyDKushBjV7-opkYm*m2jWzSgMBAS/ddd.png?size=64&crop=1%3A1

Permalink Reply by + M.Tariq Malik on April 1, 2012 at 8:31pm

(c)The purpose of cluster sampling is to sample economically while retaining the


characteristics of a probability sample. At the same time, cluster sampling addresses
two
major problems. Discuss those problems.
Research Method (STA 630) Success Objectives
Solution (c):
The purpose of cluster sampling is to sample economically while retaining the
characteristics of a
probability sample. At the same time, Cluster sampling addresses following two
problems:
1. Researchers lack a good sampling frame for a dispersed population and the cost to
reach

a sampled element is very high. A cluster is unit that contains final sampling elements but
can be treated temporarily as a sampling element itself. A researcher first samples
cluster, each of which contains elements, then draws a second sample from within the
clusters selected in the first stage of sampling. In other words, the researcher randomly
samples clusters, and then randomly samples elements from within the selected clusters.
He or she can create a good sampling frame of clusters, even if it is impossible to create
one for sampling elements. Once the researcher gets a sample of clusters, creating a
sampling frame for elements within each cluster becomes more manageable.
2. A second advantage for geographically dispersed populations is that elements within
each cluster are physically closer to each other. This may produce a savings in locating
or reaching each element.

Q: Briefly discuss the steps that are involved in conducting an experiment.


Solution:
Steps in Conducting an Experiment
Broadly there are about 12 steps in conducting an experiment, which are as below:
1. Make a hypothesis that is appropriate for experimental research.
2. Decide about an experimental design that will test the hypothesis within practical
limitations.
3. Decide how to create a situation that induces the independent variable.
4. Develop a valid and reliable measure of the dependent variable.
5. Set up an experimental setting and conduct a pilot test of the treatment and dependent
variable measures.
6. Locate appropriate subjects or cases.
7. Randomly assign subjects to groups and give careful instructions.
8. Gather data for the pretest measure of the dependent variable for all groups.
9. Introduce the treatment to the experimental group only (or to the relevant groups if
there
are multiple experimental groups) and monitor all groups.
10. Gather data for posttest measure of the dependent variable.
11. Debrief the subjects by informing them of the true purpose and reasons for the
experiment. Ask subjects what they thought was occurring.
12. Examine data collected and make comparisons between different groups. Where

appropriate, use statistics and graphs to determine whether or not the hypothesis is
supported.
Q. Classify this research on the basis of:
i. Purpose of doing research.
ii. The use of research
iii. Time dimension in research
Justify your answer.
Answer:
i. It is an exploratory research. Because it was a new issue and no research was
made on it before. The researchers had limited amount of information on this issue
and this research provided bases for many other studies.
ii. It is a basic research because it provides knowledge generated to understand a
phenomenon of interest to the researcher.
iii. It is a longitudinal research because it examines the employees at more than one
time. The research is spread upon 1927 to 1932. A group of 6 women was selected
for 5 years so it can be further classified as Panel study.
Research Method (STA 630) Success Objectives
Q. Identify these variables used in the experiment:
i. Dependent variables
ii. Independent variables
iii. Extraneous variables
Justify your answer.
Answer:
i. The variable that is the result of another variable(s) is called a dependent variable.
Productivity of the employees is the dependent variable in this case. Because it
depends on many other independent variables.
ii. The variable that causes the dependent variable is called independent variable. It is
a cause variable. In our example light illumination, rest breaks, duration of work
hours, temperature, food, wages and humidity are independent variables because
they affect productivity.
iii. Extraneous variables are factors other than the independent variable that could result
in changes in the dependent variable. In an experiment, these factors should be
controlled so that they are constant in all conditions. If they are not well-controlled,

then they result in possible alternative explanations (other than changes in the
independent variable) that could account for the changes observed in the dependent
variable. Possible extraneous variables in this research can be knowledge of the
employees that they are being observed (when employees know that they are
being watched, they tend to modify their behavior), special attention paid to them,
supervisors style, etc.
e that Xsawpmit of what he or she can study.

image: http://api.ning.com/files/Y8bD62HksPhEE7cCSUEuJu-WWaHFBRwNPfyy1UtYNOyNrQktIZ2AMYWpS9eMY97XjvEcA-6xr70r-3y0coSYoP07EKp3Ykim/1075850909.png?xgip=0%3A0%3A189%3A189%3B
%3B&width=48&height=48&crop=1%3A1

image: http://api.ning.com/files/bR8nA-I0SJG6CIZy-G9bFxGXvn7hyT4N02K91fZsBDwSAIVAhU74oiWF5BLjEKrNuuwmyDKushBjV7-opkYm*m2jWzSgMBAS/ddd.png?size=64&crop=1%3A1

Permalink Reply by + M.Tariq Malik on April 1, 2012 at 8:32pm

Q: What do you understand by the term concepts are an abstraction of reality?


Explain
with the help of an example.
Answer: Concepts are the terms and terminologies used by researchers as means of
communication. They are expressed in words, letters, signs and symbols that refer to
various
events or objects. Abstraction simply means generalization. In research methods, we
use this
term concepts are an abstraction of reality. It means that we pick up concepts from
observed
reality and goes on to the abstraction level. It is a level at which objects and events are
expressed
in more generalized form.
For example Toyota is a concept, its abstraction is a car and its further abstraction would
be
motor vehicles. We have changed the simple concept (Toyota) into an abstraction (motor
vehicles) which is a more generalized term.
Q: Join the options in column A with appropriate options given in column B.
Identify the
variables and give reasons for your choice.
Column A Column B

(a) Two men and three women were


seriously injured in a road accident 25
km away from Lahore.
(i) Intervening variable
(b) Out break of Malaria in the city
increased the consumption of mosquito
repellents.
(ii) Independent and dependent variable
(c) Distance learning system increased the
literacy level in the country.
(iii)Continuous and discontinuous variable
Answer:
Column A Column B
(a) (iii)
(b) (ii)
(c) (i)
Research Method (STA 630) Success Objectives
Identification of Variables:
(a) Men and Women are the example of discontinuous variables and the distance of 25
km is
an example of continuous variable.
(b) Out break of malaria is the independent variable and increase in the purchase of
mosquito repellents is dependent variable.
(c) There is a clear indication of many intervening variable such as flexible timing, low
fees,
low transportation cost, easy access to get education especially for far off areas.
Reasons:
(a) Discontinuous variable has limited number of distinct values which can not be
divided into
fractions, so male/female or men/women are discontinuous variables. Continuous
variable can take infinite number of values and can be divided into fractions, so the
distance is continuous variable as it can take any value e.g. 35km, 55.5 km etc.
(b) Independent variables are those variables which identifies forces or conditions that act
on

something. Malaria is the independent variable as it is causing change in other variable.


Dependent variables are the effect or result of another variable. Their occurrence is
dependent upon the occurrence of another variable. Increase in the consumption of
mosquito repellents is an outcome of increasingly spreading Malaria disease.
(c) Intervening variables are those variables that come between independent and
dependent
variables and shows a link or mechanism between them. Distance learning (independent
variable) has increased the literacy level (dependent variable) and there are other
intervening variables which are supporting this relationship e.g. flexible timing, low fees,
low transportation cost, easy access to get education especially for far off areas.
What is a grand theory?
a) One that was proposed by one of the major theorists in the sociological
tradition
b) One that is highly abstract and makes broad generalizations about the
social world
c) An intermediate level explanation of observed regularities
d) A particularly satisfactory theory that makes the researcher feel happy
What does an empiricist believe?
a) We should not apply natural science methods to social science research
b) It is the sociologists aim to understand the meaning of social action
c) Knowledge, in the form of facts, should be gained through sensory
experience
d) Research conducted within the British empire was biased and unreliable
An inductive theory is one that:
a) Involves testing an explicitly defined hypothesis
b) Does not allow for findings to feed back into the stock of knowledge
c) Uses quantitative methods whenever possible
d) Allows theory to emerge out of the data
What is the epistemological position held by a positivist?
a) There is no substitute for an indepth, hermeneutic understanding of
society
b) Scientific research should be based on valuefree, empirical observations
c) Events and discourses in the social world prevent us from having direct

knowledge of the natural order


d) It is important to remain optimistic about our research, even when things
go wrong
The interpretivist view of the social sciences is that:
a) Their subject matter is fundamentally different to that of the natural
sciences
b) We should aim to achieve the interpretive understanding of social action
c) It is important to study the way people make sense of their everyday
worlds
d) All of the above
Which of the following is an ontological question?
a) Should I use questionnaires or interviews in my project?
b) What can (and should) be considered acceptable forms of knowledge?
c) How long is it since I last visited the dentist?
d) Do social entities have an objective reality, external to social actors?
The constructionist ontological position suggests that:
a) Social phenomena and their meanings are constantly being accomplished
by social actors
b) Individuals are born into a world of rules and structures that they cannot
change
c) Building and construction work presents an ideal opportunity to exercise
the sociological imagination
d) Social facts and objects have an external reality, independently of the
people who perceive them
The qualitative research strategy places a value on:
a) Using numbers, measurements and statistical techniques
b) Generating theories through inductive research about social meanings
c) Conducting research that is of a very high quality
d) All of the above

image: http://api.ning.com/files/Y8bD62HksPhEE7cCSUEuJu-WWaHFBRwNPfyy1UtYNOyNrQktIZ2AMYWpS9eMY97XjvEcA-6xr70r-3y0coSYoP07EKp3Ykim/1075850909.png?xgip=0%3A0%3A189%3A189%3B
%3B&width=48&height=48&crop=1%3A1

image: http://api.ning.com/files/bR8nA-I0SJG6CIZy-G9bFxGXvn7hyT4N02K91fZsBDwSAIVAhU74oiWF5BLjEKrNuuwmyDKushBjV7-opkYm*m2jWzSgMBAS/ddd.png?size=64&crop=1%3A1

Permalink Reply by + M.Tariq Malik on April 1, 2012 at 8:32pm

Which of the following is an example of valuefree research?


a) Conscious partiality
b) Sympathy for the underdog
c) Unstructured interviewing
d) None of the above
An important practical issue to consider when designing a research project is:
a) Which theoretical perspective you find most interesting
b) Whether or not you have time to retile the bathroom first
c) How much time and money you have to conduct the research
d) Which colour of ring binder to present your work in
Why do you need to review the existing literature?
a) To make sure you have a long list of references
b) Because without it, you could never reach the required wordcount
c) To find out what is already known about your area of interest
d) To help in your general studying
To read critically means:
a) Taking an opposing point of view to the ideas and opinions expressed
b) Skimming through the material because most of it is just padding
c) Evaluating what you read in terms of your own research questions
d) Being negative about something before you read it
Which two of the following are legitimate frameworks for setting out a literature
review: 1. Constructing intertextual coherence, 2. Deconstruction of textual
coherence, 3. Problematizing the situation, 4. Resolving discovered problems?
a) 1 and 2
b) 2 and 3
c) 1 and 3
d) 2 and 4
A systematic literature review is:
a) One which starts in your own library, then goes to online databases and,
finally, to the internet
b) A replicable, scientific and transparent process
c) One which gives equal attention to the principal contributors to the area

d) A responsible, professional process of timemanagement for research


What is metaanalysis?
a) A technique of correcting for the errors in individual studies within a
survey of a large number of studies, to demonstrate the effect of a particular
variable
b) A process of secondarydata gathering to assemble all the possibilities for
a variables effects
c) A substitute for original research, which is justified by constraints of time
or money
d) A specialized step in a computer software program (SPSS e.g.)
What is metaethnography?
a) A technique for reviewing literature based exclusively on ethnographic
studies
b) A technique for synthesizing interpretations drawn from a number of
separate qualitative studies of the same phenomena
c) A process used to make generalizations from a range of qualitative studies
d) A process of surveying only that literature contained within a single
library
What is a narrative literature review?
a) An historicallybased review, starting with the earliest contributions to the
field
b) A review based exclusively on stories about companies, in book and casestudy
form
c) A paraphrase style of reviewing which does not require referencing
d) An initial impression of the topic which you will understand more fully as
you conduct your research
When accessing the internet, which of these steps is the most essential?
a) Recording the full URL
b) Noting the access dates
c) Downloading material to be referenced
d) They are all equally important

According to the Harvard referencing convention, which is the correct reference?


a) Bryman, A. (2008, 3e) Social Research Methods, Oxford; Oxford University
Press
b) Bryman (2008, second edition), Oxford University Press
c) Bryman, Alan, Social Research Methods (2008: OUP)
d) Bryman, A. Social Research Methods (2008)
Which of the following statements about plagiarism is most accurate?
a) It is so easy to copy and paste from the internet that everyone does it
nowadays. If a proper reference is given, where is the harm in that?
b) How can we say for sure where our own ideas come from exactly? If we
tried to give a reference for everything we could never hope to succeed.
c) Any suggestion that we have written what another actually wrote is
morally wrong. Anyway, the whole point of a literature review is to show what
we have read and what we thought about it.
d) Plagiarism is such an awful crime that those found guilty should be
obliged to wear a scarlet P on their clothing
What is a research design?
a) A way of conducting research that is not grounded in theory
b) The choice between using qualitative or quantitative methods
c) The style in which you present your research findings, e.g. a graph
d) A framework for every stage of the collection and analysis of data
If a study is reliable, this means that:
a) It was conducted by a reputable researcher who can be trusted
b) The measures devised for concepts are stable on different occasions
c) The findings can be generalized to other social settings
d) The methods are stated clearly enough for the research to be replicated
Internal validity refers to:
a) Whether or not there is really a causal relationship between two variables
b) Whether or not the findings are relevant to the participants everyday lives
c) The degree to which the researcher feels that this was a worthwhile project

d) How accurately the measurements represent underlying concepts


Lincoln & Guba (1985) propose that an alternative criterion for evaluating
qualitative research would be:
a) Impressiveness
b) Trustworthiness
c) Joyfulness
d) Messiness
Naturalism has been defined as:
a) Viewing natural and social objects as belonging to the same realm
b) Being true to the nature of the phenomenon under investigation
c) Minimising the intrusion of artificial methods of data collection into the
field
d) All of the above
In an experimental design, the dependent variable is:
a) The one that is not manipulated and in which any changes are observed
b) The one that is manipulated in order to observe any effects on the other
c) A measure of the extent to which personal values affect research
d) An ambiguous concept whose meaning depends on how it is defined

image: http://api.ning.com/files/Y8bD62HksPhEE7cCSUEuJu-WWaHFBRwNPfyy1UtYNOyNrQktIZ2AMYWpS9eMY97XjvEcA-6xr70r-3y0coSYoP07EKp3Ykim/1075850909.png?xgip=0%3A0%3A189%3A189%3B
%3B&width=48&height=48&crop=1%3A1

image: http://api.ning.com/files/bR8nA-I0SJG6CIZy-G9bFxGXvn7hyT4N02K91fZsBDwSAIVAhU74oiWF5BLjEKrNuuwmyDKushBjV7-opkYm*m2jWzSgMBAS/ddd.png?size=64&crop=1%3A1

Permalink Reply by + M.Tariq Malik on April 1, 2012 at 8:32pm

What is a crosssectional design?


a) A study of one particular section of society, e.g. the middle classes
b) One that is devised when the researcher is in a bad mood
c) The collection of data from more than one case at one moment in time
d) A comparison of two or more variables over a long period of time
Survey research is crosssectional and therefore:

a) High in replicability but low in internal validity


b) High in internal validity but low in reliability
c) High in ecological validity but low in external validity
d) None of the above
Panel and cohort designs differ, in that:
a) Cohort studies involve quantitative research, whereas panel studies are
qualitative
b) A panel study does not need rules to handle new entrants to households
c) Only a cohort study will suffer from sample attrition
d) A panel study can distinguish between age effects and cohort effects, but a
cohort design cannot
Cross cultural studies are an example of:
a) Case study design
b) Comparative design
c) Experimental design
What is rhetoric?
a) The type of rapport that is usually established in indepth interviews
b) An ancient form of poetry
c) A technique used to assess the external reliability of a data source
d) The attempt to persuade or convince an audience, often through writing
Which of the following is not usually found in a report of a quantitative study?
a) Measurement
b) Introduction
c) Confession
d) Results
The introductory section of a research report should aim to:
a) Identify the specific focus of the study
b) Provide a rationale for the dissertation, or article
c) Grab the readers attention
d) All of the above

What is the purpose of the conclusion in a research report?


a) It explains how concepts were operationally defined and measured
b) It summarizes the key findings in relation to the research questions
c) It contains a useful review of the relevant literature
d) It outlines the methodological procedures that were employed
In a report of quantitative research, an empiricist repertoire serves to:
a) Confuse the reader with long and technical words
b) Demonstrate the researchers reflexivity about their role in the research
process
c) Give the impression that the results were objective and logically inevitable
d) Provide a confessional tale of what went wrong in the procedure
Which of the following is not normally included in a written account of
qualitative research?
a) An introduction, locating the research in its theoretical context
b) An explanation of the design of the study
c) A discussion of the main findings in relation to the research questions
d) A decision to accept or reject the hypothesis
Postmodernist theorists challenge the idea of objective truth by arguing that:
a) There are many possible ways of interpreting and representing social
reality
b) It is important to uncover the social laws that operate in an external reality
c) Only women have the unique standpoint needed to be able to make
universal truth claims
d) All of the above
Apart from postmodernism, what other intellectual trend has stimulated an
interest in the way social scientists use rhetorical devices in their writing?
a) Positivism
b) Social studies of science
c) Traditional ethnography
d) Existentialist philosophy
A reflexive social researcher will be inclined to write about:
a) The effects that their values, biases and theoretical leanings might have

had upon the data collection and analysis


b) The way in which their findings unfolded naturally and inevitably
through logical deduction
c) The way in which their findings are objectively truthful and valid
d) The unproblematic and straightforward procedures of designing research,
building a rapport with participants and interpreting the findings
The three forms of ethnographic writing that Van Maanen (1988) identifies are:
a) Positivist stories, interpretivist stories and realist stories
b) Native accounts, tourist accounts and voyeuristic accounts
c) Realist tales, confessional tales and impressionist tales
d) Feminist accounts, ethnomethodological accounts and postmodern
accounts
Which of the following is not a problem associated with using web sites as
sources of data?
a) The sample of web sites is only as good as the keywords used to search for
them
b) It is difficult to find any web sites about most topics in social research
c) New web sites are constantly appearing while others are disappearing
d) The content of web sites is likely to change as they are updated
What is distinctive about asynchronous online communication?
a) The interviewer and their respondents write at different times
b) It cannot take place on the World Wide Web
c) It occurs in real time, with participants responding to questions
immediately
d) It cannot be conducted by email
What is a virtual ethnography?
a) The use of visual data rather than written texts for content analysis
b) A technique used to facilitate online focus groups
c) A study that uses participant observation but not interviewing
d) An ethnographic study of an online community or social setting
Which of the following is a practical problem associated with asynchronous

focus groups?
a) It is difficult to send out a welcome message to participants this way
b) Moderators cannot be available online 24 hours a day
c) Not all participants will have access to the required conferencing software
d) Participants do not have enough time to write detailed responses

image: http://api.ning.com/files/Y8bD62HksPhEE7cCSUEuJu-WWaHFBRwNPfyy1UtYNOyNrQktIZ2AMYWpS9eMY97XjvEcA-6xr70r-3y0coSYoP07EKp3Ykim/1075850909.png?xgip=0%3A0%3A189%3A189%3B
%3B&width=48&height=48&crop=1%3A1

image: http://api.ning.com/files/bR8nA-I0SJG6CIZy-G9bFxGXvn7hyT4N02K91fZsBDwSAIVAhU74oiWF5BLjEKrNuuwmyDKushBjV7-opkYm*m2jWzSgMBAS/ddd.png?size=64&crop=1%3A1

Permalink Reply by + M.Tariq Malik on April 1, 2012 at 8:37pm

Question No: 16 ( Marks: 1 ) - Please choose one


In a study of concept learning in first-graders, all of the following should happen, except;
To help make the experience enjoyable, children should be given substantial
incentives to participate.
The children should be asked if they want to participate.
Teachers of the children in their classes should give informed consent.
Parents must give consent.
Question No: 17 ( Marks: 1 ) - Please choose one
Which of the following is least likely to occur as an ethical problem with electronic
research?
Privacy could be invaded.
People are not likely to be able to stop once they have begun participating.
Debriefing could be avoided.
Informed consent cannot be completely monitored.
Question No: 18 ( Marks: 1 ) - Please choose one
Quantitative research has been criticised because:
The reliance on instruments and procedures makes it high in ecological validity.
The measurement process suggests a spurious and artificial sense of accuracy.
It underestimates the similarities between objects in the natural and social worlds.

It has no validity.
Question No: 19 ( Marks: 1 ) - Please choose one
Which of the following is a characteristic of a standardized test?
The administration of the test is controlled carefully to ensure that all examinees
experience the same conditions.
The test is developed by experts to ensure it is technically sound.
The scores are interpreted in standard ways.
All of the given options
Question No: 20 ( Marks: 1 ) - Please choose one
Mr. Latif is measuring students' attitudes on a controversial topic. Which of the following
would you recommend he use to control for the potential problem of students feeling the
need to respond with socially acceptable responses?
Ensure construct validity
Allow anonymous responses
Ensure reliability
Ensure confidentiality
Question No: 21 ( Marks: 1 ) - Please choose one
Which of the following scales can measure the Temperature?
Nominal scale
Ordinal scale
Interval scale
All of the given options
Question No: 22 ( Marks: 1 ) - Please choose one
Following are the properties of nominal scale, except;
Least powerful
Suggest no order
Provide categorical information
Provide magnitude of object

Question No: 23 ( Marks: 1 ) - Please choose one


Which of the following is NOT the implication of Ordinal scale?
Characteristics of nominal scale
Rank the object
Does not provide magnitude of object
Provide rate

Question No: 24 ( Marks: 1 ) - Please choose one


Which of the following statements is correct about validity and reliability?
When internal validity is high, external validity is low
When internal validity is high, there is no change in external validity
When internal validity is high, external validity is also high
All of the given options
Question No: 25 ( Marks: 1 ) - Please choose one
When you are confident that the experimental manipulation produced the changes you
measured in the dependent variable, your study probably has good ________ validity.
Internal
External
Causal
Construct
Question No: 26 ( Marks: 1 ) - Please choose one
Mr. Ali has conducted an extensive review of the literature and has deductively reasoned
a hypothesis about his problem on the basis of this review. Which type of a research plan
is Mr. Ali likely proposing?
Qualitative
Ethical
Both quantitative and qualitative
Quantitative

Question No: 27 ( Marks: 1 ) - Please choose one


Which of the following is a legitimate purpose of a research plan?
It forces you to think through every aspect of a study.
It provides detailed procedures to guide the conduct of the study.
It provides clear roadmap of the study.
Writing a plan facilitates the evaluation of it.
Question No: 28 ( Marks: 1 ) - Please choose one
In the statement Smoke badly destroys health of people what is the unit of analysis?
Transportation
Individuals
Pollutants
Drugs
Question No: 29 ( Marks: 1 ) - Please choose one
While studying supervisor-subordinate relationship in an organization, what is the unit of
analysis?
Individual
Dyad
Employer-employee
Organization
Question No: 30 ( Marks: 1 ) - Please choose one
Which of the following types of survey research is often used for program evaluation or
accreditation purposes?
Developmental surveys
Followup studies
Public opinion polls
No survey research is appropriate
Question No: 31 ( Marks: 1 ) - Please choose one
Which of the following is NOT an example of a problem with using survey research to
investigate behaviour?
Respondents tend to answer by giving the most socially desirable answer.

Respondents may inadvertently omit key terms in the question.


Respondents may not give an honest reply to what they see as a threatening question.
Respondents may be unwilling to reveal information as they believe they are
likely to be identified.
Question No: 32 ( Marks: 1 ) - Please choose one
What should the researcher do if they have achieved a low response rate?
Copy responses and paste them again.
Abandon the research project entirely.
Recognize and accept the possible limitations of a low response rate.
Fill in some more questionnaires themselves.
Question No: 33 ( Marks: 1 ) - Please choose one
Which of the following statements is correct about questionnaire?
A questionnaire should run to at least twenty pages.
The questions should have no spaces between them so that the whole questionnaire
looks small.
Only questionnaires that are A3 size achieve high response rates.
Clear presentation is more important than overall size.
Question No: 34 ( Marks: 1 ) - Please choose one
Critical decision area in questionnaire construction is which of the following?
Question content
Question wording
Response strategy
All of the given options

image: http://api.ning.com/files/Y8bD62HksPhEE7cCSUEuJu-WWaHFBRwNPfyy1UtYNOyNrQktIZ2AMYWpS9eMY97XjvEcA-6xr70r-3y0coSYoP07EKp3Ykim/1075850909.png?xgip=0%3A0%3A189%3A189%3B
%3B&width=48&height=48&crop=1%3A1

image: http://api.ning.com/files/bR8nA-I0SJG6CIZy-G9bFxGXvn7hyT4N02K91fZsBDwSAIVAhU74oiWF5BLjEKrNuuwmyDKushBjV7-opkYm*m2jWzSgMBAS/ddd.png?size=64&crop=1%3A1

Permalink Reply by + M.Tariq Malik on April 1, 2012 at 8:38pm

Question No: 35 ( Marks: 1 ) - Please choose one


The most critical area of an article to read is;
Introduction
Abstract
Results section
Limitations

Question No: 36 ( Marks: 1 ) - Please choose one


Which of the following is/are advantage(s) of the self-completed questionnaire?
Inability to confirm who completed the questionnaire
Its unsuitability for some kinds of respondents
Inability to ask many questions that are not directly relevant to the respondent
Easy to justify the answer
Question No: 37 ( Marks: 1 ) - Please choose one
Time consumed in mall intercept interview is representing which of the following type?
High
Moderate
Low
Nill
Question No: 38 ( Marks: 1 ) - Please choose one
Which of the following similarity is found in Qualitative research and Survey research?
Examine topics primarily from the participants' perspectives.
They are guided by predetermined variables to study.
They are descriptive research methods.
Have large sample sizes.
Question No: 39 ( Marks: 1 ) - Please choose one
Which of the following represent a good qualitative problem statement?
Defines the independent and dependent variables.
conveys a sense of emerging design.

Specifies a research hypothesis to be tested.


Specifies the relationship between variables that the researcher expects to find.

Question No: 40 ( Marks: 1 ) - Please choose one


Which of the following makes research questions very crucial?
Guide your decisions about what data to collect and from where.
Help you decide which research area interests you.
Ensure that your findings have external validity.
Prevent you from thinking about research strategies.
Question No: 41 ( Marks: 10 )
How would you explain "Research Proposal" and its different sections?
1. When a extraneous variable systematically varies with the independent variable
and influences the dependent variable, it is called:
a. Another dependent variable
b. A confounding variable
c. A moderating variable
d. An unreliable variable
2. Which of the following statements is true?
a. A statistical relationship is sufficient evidence to infer causality
b. Temporal order of the cause and effect is not important in inferring causality
c. A statistical relation of X and Y is insufficient evidence for inferring
causality
d. Temporal order of cause and effect variables and statistical relation are all that
are needed to infer causality

image: http://api.ning.com/files/Y8bD62HksPhEE7cCSUEuJu-WWaHFBRwNPfyy1UtYNOyNrQktIZ2AMYWpS9eMY97XjvEcA-6xr70r-3y0coSYoP07EKp3Ykim/1075850909.png?xgip=0%3A0%3A189%3A189%3B
%3B&width=48&height=48&crop=1%3A1

image: http://api.ning.com/files/bR8nA-I0SJG6CIZy-G9bFxGXvn7hyT4N02K91fZsBDwSAIVAhU74oiWF5BLjEKrNuuwmyDKushBjV7-opkYm*m2jWzSgMBAS/ddd.png?size=64&crop=1%3A1

Permalink Reply by + M.Tariq Malik on April 1, 2012 at 8:38pm

3. A school district examines a program that uses mentors to help very poor
readers improve their reading performance. The children in the program are at the
4th percentile at pretest. At posttest they are around the 20th percentile. While it is
possible that the program made the difference, another reason for the change in
scores could be:
a. History
b. Regression artifact
c. Multiple-treatment interference
d. Differential selection
4. A group of researchers do a study where children from particular classrooms
are assigned to treatment or control conditions. After the study, the researcher
finds out that the students in the control group are higher achievers than those in
the experimental group. He found no treatment effect. The failure to find an effect
may be due to:
a. A treatment effect
b. A testing effect
c. A differential selection effect
d. A maturation effect
5. A researcher examines a program looking at the effects of mentoring on poor
readers' reading achievement. He looks at two different schools. One serves as the
control and the other the experimental group. Both schools had reading
achievement that was around the 50th percentile. During the time that the
mentoring program is in place in the experimental group, a statewide reading
initiative is started in randomly selected schools. The experimental, but not the
control school is involved in the initiative. At the end of the year, the
experimental group does better than the control. From the information presented

above, a likely threat to the internal validity of the study is:


a. Selection by mortality interaction
b. Mortality
c. Selection-history effect
d. Selection-maturation effect
6. Which type of validity refers to the degree to which you can infer that the
relationship between two variables is causal?
a. Internal validity
b. Population validity
c. Ecological validity
d. Statistical conclusion validity
7. Which type of validity refers to the ability to infer that the independent and
dependent variables are related ant that the measured strength of the relationship
is accurate?
a. Internal validity
b. Population validity
c. Ecological validity
d. Statistical conclusion validity
8. An extraneous variable that systematically varies with the independent variable
and also influences the dependent variable is known as a _______________.
a Confounding variable
b. Third variable
c. Second variable
d. Both a and b are correct
9. The use of multiple observers to allow cross-checking of observations to make
sure that the investigators agree with what took place is known as _______.
a. Interpretive validity
b. Researcher bias
c. Multiple operationalism

d. Investigator triangulation
10. _____________ is the lowest inference descriptor of all because it uses the
participants own words.
a. Participant feedback
b. A verbatim
c. Data triangulation
d. Investigator triangulation
11. ___________ refers to physical or mental changes that may occur within
individuals over time, such as aging, learning, boredom, hunger, and fatigue.
a. Instrumentation
b. History
c. Maturation
d. Testing
12. What type of validity refers to the extent to which the results of a study can be
generalized across time?
a. Ecological validity
b. External validity
c. Internal validity
d. Temporal validity
13. Which of the following best describes interpretive validity?
a. Factual accuracy of an account as reported by the researcher
b. Accurately portraying the meanings given by the participants to what is
being studied
c. Degree to which a theoretical explanation fits the data
d. Ability to generalize the study results across settings

14. Which of the following terms is a strategy where the researcher actively

engages in critical self-reflection about his or her potential biases and


predispositions.
a. Experimenter effect
b. Reactivity
c. Investigator triangulation
d. Reflexivity
15. Which of the following is not considered one of the criteria for inferring
causality?
a. Evidence that the independent and dependent variables are related
b. Evidence that the relationship between the variables being investigated is not
due to a
confounding extraneous variable
c. Evidence that changes in variable A occur before changes in variable B
d. The temporal ordering of the variables being investigated does not matter
because a relationship is all that is really needed
16. The use of multiple data sources to help understand a phenomenon is one
strategy that is used to promote qualitative research validity. Which of the
following terms describes this strategy?
a. Data matching
b. Pattern matching
c. Data triangulation
d. Data feedback

17. What may happen when different comparison groups experience a different
history event?
a. History effect

b. Selection-history effect
c. Selection effect
d. Group effect
18. What is another term that refers to a confounding extraneous variable?
a. Last variable
b. First variable
c. Third variable
d. Fourth variable
19. Which of the following refers to any systematic change that occurs over time
in the way in which the dependent variable is assessed?
a. Instrumentation
b. Maturation
c. Testing
d. Selection
20. Which of the following terms describes the ability to generalize from the
sample of individuals on which a study was conducted to the larger target
population of individuals and across different subpopulations within the larger
target population?
a. External validity
b. Population validity
c. Ecological validity
d. Temporal validity

image: http://api.ning.com/files/Y8bD62HksPhEE7cCSUEuJu-WWaHFBRwNPfyy1UtYNOyNrQktIZ2AMYWpS9eMY97XjvEcA-6xr70r-3y0coSYoP07EKp3Ykim/1075850909.png?xgip=0%3A0%3A189%3A189%3B
%3B&width=48&height=48&crop=1%3A1

image: http://api.ning.com/files/bR8nA-I0SJG6CIZy-G9bFxGXvn7hyT4N02K91fZsBDwSAIVAhU74oiWF5BLjEKrNuuwmyDKushBjV7-opkYm*m2jWzSgMBAS/ddd.png?size=64&crop=1%3A1

Permalink Reply by + M.Tariq Malik on April 1, 2012 at 8:39pm

21. Which of the following is not a strategy used to promote qualitative research

validity?
a. Peer review
b. Theory triangulation
c. Extended fieldwork
d. Random assignment
22. The use of several measures of a construct is called:
a. Multiple operationalism
b. Multiple construct measurement
c. Operationalism
d. Methods triangulation
23. A physical or mental change that occurs in participants over time that affects
their performance on the dependent variable is called ________.
a. Instrumentation
b. Maturation
c. Regression
d. None of above
24. Attrition generally occurs in research where ____.
a. You do demographic research
b. The study fails
c. Some participants do not complete the study
d. The study is very brief

25. Differential attrition occurs when the people dropping out from one group are
different from the others in their group or from the people in the comparison
group.

a. True
b. False
26. Internal validity refers to which of the following?
a. The ability to infer that a casual relationship exists between 2 variables
b. The extent to which study results can be generalized to and across populations
of persons,
settings, and times
c. The use of effective measurement instruments in the study
d. The ability to generalize the study results to individuals not included in the
study
27. Which strategy used to promote qualitative research validity uses multiple
research methods to study a phenomenon?
a. Data triangulation
b. Methods triangulation
c. Theory triangulation
d. Member checking
28. Which type of validity refers to the factual accuracy of an account as reported
by the researcher?
a. Ecological validity
b. Temporal validity
c. Descriptive validity
d. None of the above

29. Which of the following in not one of the key threats to internal validity?
a. Maturation
b. Instrumentation
c. Temporal change
d. History

30. This type of validity refers to the ability to generalize the results of a study
across settings.
a. Temporal validity
b. Internal validity
c. Ecological validity
d. External validity
31. Which is not a direct threat to the internal validity of a research design?
a. History
b. Testing
c. Sampling error
d. Differential selection
32. Alteration in performance due to being aware that one is participating in a
study is known as ______.
a. Operationalism
b. Reactivity
c. Temporal validity
d. Mortality

33. The idea that the more times a research finding is shown with different sets of
people, the more confidence we can place in the finding and in generalizing
beyond the original participants is known as ___________.
a. Naturalistic generalization
b. Methods generalization
c. Data triangulation
d. Replication logic

The scientific method is preferred over other ways of knowing because it is more;
Reliable
Systematic
Accurate
All of the given options
An operational definition is:
One that bears no relation to the underlying concept.
An abstract, theoretical definition of a concept.
A definition of a concept in terms of specific, empirical measures.
One that refers to opera singers and their work.
If a researcher is studying the effect of using laptops in his classroom to ascertain
their merit and worth, he is likely conducting which type of research?
Basic
Applied
Evaluation
Experimental

What is the basis of the Scientific Method?


To test hypotheses in conditions those are condusive to its success.
To formulate a research problem and disprove the hypothesis.
To formulate a research problem, test the hypothesis in carefully controlled
conditions that challenge the hypothesis.
To test hypotheses and if they are disproved, they should be abandoned completely.
Which of the following is a concept?
Leadership.

Total Quality Management.


Human Resource Management.
All of the given options
Meanings attached to a concept create problems of measurements.
Unambiguous
Vague
Clear
Dictionary
To explain, predict, and/or control phenomena are the goal of---------------------Tradition
Inductive logic
Deductive logic
The scientific
Why, as scientists, do we not want to rely on authority for explanations?
Those in authority are often wrong.
Those in authority cannot be challenged.
Those in authority rely too much on objective information.
Those in authority often have no common sense.
If a researcher was studying the use of various instructional approaches to the
"multiple intelligences" of his students, he is likely to be conducting which type of
research?
Basic
Applied
Evaluation
Grounded theory
A concept is a generalized idea about------ .
A class of objects
Attributes

Occurrences
All of the given
Under which of the following research method is not applicable?
Health care
Business
Government offices
Imaginary worlds

image: http://api.ning.com/files/Y8bD62HksPhEE7cCSUEuJu-WWaHFBRwNPfyy1UtYNOyNrQktIZ2AMYWpS9eMY97XjvEcA-6xr70r-3y0coSYoP07EKp3Ykim/1075850909.png?xgip=0%3A0%3A189%3A189%3B
%3B&width=48&height=48&crop=1%3A1

image: http://api.ning.com/files/bR8nA-I0SJG6CIZy-G9bFxGXvn7hyT4N02K91fZsBDwSAIVAhU74oiWF5BLjEKrNuuwmyDKushBjV7-opkYm*m2jWzSgMBAS/ddd.png?size=64&crop=1%3A1

Permalink Reply by + M.Tariq Malik on April 1, 2012 at 8:39pm

http://.com

1- Hypothesis refers to
A. The outcome of an experiment
B. A conclusion drawn from an experiment
C. A form of bias in which the subject tries to outguess the experimenter
D. A tentative statement about the relationship
2- Statistics is used by researchers to
A. Analyze the empirical data collected in a study
B. Make their findings sound better
C. Operationally define their variables
D. Ensure the study comes out the way it was intended
3- A literature review requires
A. Planning
B. Good & clear writing
C. Lot of rewriting

D. All of the above


4- A literature review is based on the assumption that
A. Copy from the work of others
B. Knowledge accumulates and learns from the work of others
C. Knowledge disaccumulates
D. None of the above option

5- A theoretical framework
A. Elaborates the r/s among the variables
B. Explains the logic underlying these r/s
C. Describes the nature and direction of the r/s
D. All of the above
6- Which of the following statement is not true?
A. A research proposal is a document that presents a plan for a project
B. A research proposal shows that the researcher is capable of successfully
conducting the proposed research project
C. A research proposal is an unorganized and unplanned project
D. A research proposal is just like a research report and written before the
research project
7- Preliminary data collection is a part of the
A. Descriptive research
B. Exploratory research
C. Applied research
D. Explanatory research
8- Conducting surveys is the most common method of generating
A. Primary data
B. Secondary data
C. Qualitative data

D. None of the above

9- After identifying the important variables and establishing the logical reasoning in
theoretical framework, the next step in the research process is
A. To conduct surveys
B. To generate the hypothesis
C. To focus group discussions
D. To use experiments in an investigation
10- The appropriate analytical technique is determined by
A. The research design
B. Nature of the data collected
C. Nature of the hypothesis
D. Both A & B
E. Personal interviews conducted in shopping malls are known as:
a. Mall interviews
b. Mall intercept interviews
c. Brief interviews
d. None of the given options
F. WATS lines provided by long distance telephone service at fixed rates.
In this regard, WATS is the abbreviation of:
a. West Africa Theological Seminary
b. Washtenaw Area Transportation Study
c. Wide Area Telecommunications Service
d. World Air Transport Statistics

G. A list of questions which is handed over to the respondent, who reads


the questions and records the answers himself is known as the:
a. Interview schedule
b. Questionnaire
c. Interview guide
d. All of the given options
H. One of the most critical stages in the survey research process is:
a. Research design
b. Questionnaire design
c. Interview design
d. Survey design
I. Question that consists of two or more questions joined together is
called a:
a. Double barreled question
b. General question
c. Accurate question
d. Confusing question
J. The number of questionnaires returned or completed divided by the
total number of eligible people who were contacted or asked to
participate in the survey is called the:
a. Response rate
b. Participation rate
c. Inflation rate
d. None of the given options

http://.com
K. To obtain the freest opinion of the respondent, when we ask general
question before a specific question then this procedure is called as the:
a. Research technique
b. Qualitative technique
c. Funnel technique
d. Quantitative technique
L. A small scale trial run of a particular component is known as:
a. Pilot testing
b. Pre-testing
c. Lab experiments
d. Both A & B
M. Field testing of the questionnaire shows that:
a. Respondents are willing to co-operate
b. Respondents are not willing to co-operate
c. Respondents do not like any participation
d. All of the given options
N. Service evaluation of hotels and restaurants can be done by the:
a. Self-administered questionnaires
b. Office assistant
c. Manager
d. None of the given options
A.

image: http://api.ning.com/files/Y8bD62HksPhEE7cCSUEuJu-WWaHFBRwNPfyy1UtYNOyNrQktIZ2AMYWpS9eMY97XjvEcA-6xr70r-3y0coSYoP07EKp3Ykim/1075850909.png?xgip=0%3A0%3A189%3A189%3B
%3B&width=48&height=48&crop=1%3A1

image: http://api.ning.com/files/bR8nA-I0SJG6CIZy-G9bFxGXvn7hyT4N02K91fZsBDwSAIVAhU74oiWF5BLjEKrNuuwmyDKushBjV7-opkYm*m2jWzSgMBAS/ddd.png?size=64&crop=1%3A1

Permalink Reply by + M.Tariq Malik on April 1, 2012 at 8:39pm

1. Which one of the following sets is the measure of central tendency?


a. Mean, standard deviation, mode
b. Mean, median, standard deviation
c. Arithmetic mean, median, mode
d. Standard deviation, internal validity, mode
2. In lab experiment the effect of Variables is controlled to evaluate the
causal relationship.
a. Extraneous
b. Moderate
c. Intervening
d. All of the above
3. Internal validity refers to .
a. Researchers degree of confidence.
b. Generalizability
c. Operationalization
d. All of the above
4. Which of the following is the weakest experimental design?
a. One group pretest-posttest design
b. Quasi- experimental design
c. Two group posttest only design
d. Ex post facto design
5. How many times the students appear in the research class is the example of
_________.
a. Intensity
b. Space
c. Frequency
d. Direction
6. Disadvantage of content analysis is .

a. Researcher can increase the sample size


b. Provides access on the subjects to which researcher does have physical access.
c. Sometime documents provide incomplete account to the researcher
d. Spontaneous feelings can be recorded when they occurred
7. Which of the following statement is incorrect with respect to An
experimental design is a set of procedures specifying:
a. How the test units (subjects) are to be divided into homogenous sub samples.
b. What independent variables or treatments are to be measured?
c. What dependent variables are to be measured?
d. How the extraneous variables are to be controlled?
8. Time consumed in mall intercept interview is .
a. High
b. Moderate
c. Low
d. Nil
9. Teacher should create a friendly environment in the classroom this is the
type of .
a. Leading question
b. Loaded question
c. Double Barreled
d. Burdensome question

10. Departmental stores selected to test a new merchandising display system is


the example of .
a. Quota sampling
b. Convenience sampling
c. Judgmental sampling
d. Purposive sampling

1. Discrete variable is also called.


A. Categorical variable
B. Discontinuous variable
C. Both A & B
D. None of the above
2. Officers in my organization have higher than average level of commitment
such a hypothesis is an example of.
A. Descriptive Hypothesis
B. Directional Hypothesis
C. Relational Hypothesis
D. All of the above
3. Science refers to.
A. A system for producing knowledge
B. The knowledge produced by a system
C. Both A & B
D. None of the above
4. Which one of the following is not a characteristic of scientific method?
A. Deterministic
B. Rationalism
C. Empirical
D. Abstraction
5. The theoretical framework discusses the interrelationships among the.
A. Variables
B. Hypothesis
C. Concept
D. Theory
1. research is based on naturalism.
A. Field research
B. Descriptive research
C. Basic research

D. Applied research
2. Personal interviews conducted in shopping malls are known as
A. Mall interviews
B. Mall intercept interviews
C. Brief interviews
D. None of the given options
3. is used to obtain the freest opinion of the respondent, by asking
general question before a specific question.
A. Research technique
B. Qualitative technique
C. Funnel technique
D. Quantitative technique
4. In, the interviewer and members jointly control the pace and
direction of the interview.
A. Field interview
B. Telephonic interview
C. Both A and B
D. None of the given options
5. Randomization of test units is a part of
A. Pretest
B. Posttest
C. Matching
D. Experiment
http://.com

Rationalism is the application of:


Select correct option:
Research solution

Logic and arguments


Reasoning
Previous findings

image: http://api.ning.com/files/Y8bD62HksPhEE7cCSUEuJu-WWaHFBRwNPfyy1UtYNOyNrQktIZ2AMYWpS9eMY97XjvEcA-6xr70r-3y0coSYoP07EKp3Ykim/1075850909.png?xgip=0%3A0%3A189%3A189%3B
%3B&width=48&height=48&crop=1%3A1

image: http://api.ning.com/files/bR8nA-I0SJG6CIZy-G9bFxGXvn7hyT4N02K91fZsBDwSAIVAhU74oiWF5BLjEKrNuuwmyDKushBjV7-opkYm*m2jWzSgMBAS/ddd.png?size=64&crop=1%3A1

Permalink Reply by + M.Tariq Malik on April 1, 2012 at 8:40pm

1- Hypothesis refers to
A. The outcome of an experiment
B. A conclusion drawn from an experiment
C. A form of bias in which the subject tries to outguess the experimenter
D. A tentative statement about the relationship
2- Statistics is used by researchers to
A. Analyze the empirical data collected in a study
B. Make their findings sound better
C. Operationally define their variables
D. Ensure the study comes out the way it was intended
3- A literature review requires
A. Planning
B. Good & clear writing
C. Lot of rewriting
D. All of the above
4- A literature review is based on the assumption that
A. Copy from the work of others
B. Knowledge accumulates and learns from the work of others
C. Knowledge disaccumulates
D. None of the above option

5- A theoretical framework
A. Elaborates the r/s among the variables
B. Explains the logic underlying these r/s
C. Describes the nature and direction of the r/s
D. All of the above

6- Which of the following statement is not true?


A. A research proposal is a document that presents a plan for a project
B. A research proposal shows that the researcher is capable of successfully
conducting the proposed research project
C. A research proposal is an unorganized and unplanned project
D. A research proposal is just like a research report and written before the research
project
7- Preliminary data collection is a part of the
A. Descriptive research
B. Exploratory research
C. Applied research
D. Explanatory research
8- Conducting surveys is the most common method of generating
A. Primary data
B. Secondary data
C. Qualitative data
D. None of the above
9- After identifying the important variables and establishing the logical reasoning in
theoretical framework, the next step in the research process is
A. To conduct surveys
B. To generate the hypothesis
C. To focus group discussions

D. To use experiments in an investigation


10- The appropriate analytical technique is determined by
A. The research design
B. Nature of the data collected
C. Nature of the hypothesis
D. Both A & B

E. Personal interviews conducted in shopping malls are known as:


1. Mall interviews
2. Mall intercept interviews
3. Brief interviews
4. None of the given options
F. WATS lines provided by long distance telephone service at fixed rates. In this
regard, WATS is the abbreviation of:
1. West Africa Theological Seminary
2. Washtenaw Area Transportation Study
3. Wide Area Telecommunications Service
4. World Air Transport Statistics
G. A list of questions which is handed over to the respondent, who reads the
questions and records the answers himself is known as the:
1. Interview schedule
2. Questionnaire
3. Interview guide
4. All of the given options
H. One of the most critical stages in the survey research process is:

1. Research design
2. Questionnaire design
3. Interview design
4. Survey design
I. Question that consists of two or more questions joined together is called a:
1. Double barreled question
2. General question
3. Accurate question
4. Confusing question

J. The number of questionnaires returned or completed divided by the total number


of eligible people who were contacted or asked to participate in the survey is called
the:
1. Response rate
2. Participation rate
3. Inflation rate
4. None of the given options
K. To obtain the freest opinion of the respondent, when we ask general question
before a specific question then this procedure is called as the:
1. Research technique
2. Qualitative technique
3. Funnel technique
4. Quantitative technique
L. A small scale trial run of a particular component is known as:
1. Pilot testing

2. Pre-testing
3. Lab experiments
4. Both A & B
M. Field testing of the questionnaire shows that:
1. Respondents are willing to co-operate
2. Respondents are not willing to co-operate
3. Respondents do not like any participation
4. All of the given options
N. Service evaluation of hotels and restaurants can be done by the:
1. Self-administered questionnaires
2. Office assistant
3. Manager
4. None of the given options

1-The researcher protects the confidentiality in following ways, Except;


Select correct option:
Obtaining signed nondisclosure documents.
Restricting access to data instruments where the respondent is identified.
Disclosure of data subsets.
Restricting access to respondent identification.
2-Which of the following should not be a consideration in writing a proposal?
Select correct option:
Understanding the problem situation
The appearance/form of the proposal
Responding to every element exactly as specified by the client
Knowing as much as possible about the proposal recipients

3-Under which of the following research method is not applicable?


Select correct option:
Health care
Business
Government offices
Imaginary worlds
4-Which of the following is a concept?
Select correct option:
Leadership.
Total Quality Management.
Human Resource Management.
All of the given options
5-In which way does a ratio scale measurement differ from an interval measurement?
Select correct option:
It measures larger or smaller scores on some underlying dimension.
It implies the existence of an absolute zero value.
It requires the mutual exclusivity of all cases.
It requires that exhaustiveness is applied to all observations.

6-If a researcher was studying the use of various instructional approaches to the "multiple
intelligences" of his students, he is likely to be conducting which type of research?
Select correct option:
Basic
Applied
Evaluation
Grounded theory
applieed

7-An operational definition is:


Select correct option:
One that bears no relation to the underlying concept.
An abstract, theoretical definition of a concept.
A definition of a concept in terms of specific, empirical measures.
One that refers to opera singers and their work.
8-In the statement 'an examination of the effects of discretionary behaviour on employee
performance' employee performance is the:
Select correct option:
Independent variable
Dependent variable
Extraneous variable
Causal variable
9-Quantitative researcher's preoccupation with generalization is an attempt to:
Select correct option:
Develop the law like findings of the natural sciences.
Boost their chances of publication.
Enhance the internal validity of their research.
Demonstrate the complex techniques of statistical analysis.
10-Which one of the following tasks is NOT accomplished by theoretical framework?
Select correct option:
Elaborate the relationship among the variables
Explain the logic underlying the relationship between the variables.
Describe the nature, and direction of the relationships among the variables.
Relates the previous studies.
11-What is required if there is a chance that data could harm the respondents?
Select correct option:
Change of results
A signed form detailing the types of limits

Free consent of respondents


All of the given options
12-Which of the following is not an example of deception?
Select correct option:
A part of the truth is not communicated to the respondent
To observe code of ethics
To leak out information regarding any confidential matter
To create bias between respondents before the survey
13-Which of the the following is not included in Objects?
Select correct option:
Tables
People
Books
Weight
14-The extent to which we can generalize the results of a study to other participants is
called:
Select correct option:
Sampling validity
External validity
Construct validity
Internal validity

15-If a nominal scale is used, it is permissible to calculate which of the following


statistics?
Select correct option:
Mean
Range
Percentile
Mode

16-All of the following are true about theories, except;


Select correct option:
They organize and summarize existing knowledge about a topic.
They are tentative until the facts prove them true.
They provide possible explanations for the phenomenon under study.
They lead to testable hypotheses through the process of deduction.
17-Research questions are crucial because they will:
Select correct option:
Guide your decisions about what data to collect and from where.
Help you decide which research area interests you.
Ensure that your findings have external validity.
Prevent you from thinking about research strategies.
18-What is the basis of the Scientific Method?
Select correct option:
To test hypotheses in conditions that are condusive to its success.
To formulate a research problem and disprove the hypothesis.
To formulate a research problem, test the hypothesis in carefully controlled
conditions
that challenge the hypothesis.
To test hypotheses and if they are disproved, they should be abandoned completely.
19-Procedures determining what two issues are rarely used in quantitative research?
Select correct option:
Objectivity and subjectivity
Reliability and validity
Accessibility and replicability
Quality and quantity

20-Ms. Laiba has decided to use the test at the end of the textbook to measure the
achievement
levels of the students in her study. Which of the following BEST describes the chapter
test?
Select correct option:
Definition
Construct
Variable
Operationalized variable

image: http://api.ning.com/files/Y8bD62HksPhEE7cCSUEuJu-WWaHFBRwNPfyy1UtYNOyNrQktIZ2AMYWpS9eMY97XjvEcA-6xr70r-3y0coSYoP07EKp3Ykim/1075850909.png?xgip=0%3A0%3A189%3A189%3B
%3B&width=48&height=48&crop=1%3A1

image: http://api.ning.com/files/bR8nA-I0SJG6CIZy-G9bFxGXvn7hyT4N02K91fZsBDwSAIVAhU74oiWF5BLjEKrNuuwmyDKushBjV7-opkYm*m2jWzSgMBAS/ddd.png?size=64&crop=1%3A1

Permalink Reply by + M.Tariq Malik on April 1, 2012 at 8:41pm

Quiz 01
STA 630- Research Methods
Choose the best option and give answer according to the
instructions
1. Rationalism is the application of which of the following?
A. Logic and arguments
B. Research solution
C. Reasoning
D. Previous findings
2. On which of the following, scientific knowledge mostly relies?
A. Logical understanding
B. Identification of events
C. Prior knowledge
D. All of the given options
3. Which of the following refers to research supported by measurable
evidence?
A. Opinion

B. Empiricism
C. Speculation
D. Rationalism
4. Research method is applicable in all of the following fields, EXCEPT;
A. Health care
B. Religion
C. Business
D. Government offices
5. All of the following are true statements about action research, EXCEPT;
A. Data are systematically analyzed
B. Data are collected systematically
C. Results are generalizable
D. Results are used to improve practice
6. Which of the following is characteristic of action research?
A. Variables are tightly controlled
B. Results are generalizable
C. Data are usually qualitative
D. Results demonstrate cause-and-effect relationships
7. If a researcher is studying the effect of using laptops in his classroom to
ascertain their merit and worth, he is likely conducting which of the
following types of research?
A. Experimental
B. Applied
C. Basic
D. Evaluation

8. Exploratory research addresses which of the following types of

question?
A. If
B. How
C. Why
D. What
9. Which of the following is not the source for getting information for
exploratory research?
A. Content analysis
B. Survey
C. Case study
D. Pilot study
10. Which of the following is the main quality of a good theory?
A. A theory that has survived attempts at falsification
B. A theory that is proven to be right
C. A theory that has been disproved
D. A theory that has been falsified
11. Which of the following is not a concept?
A. Leadership
B. Total Quality Management
C. Intelligence Quotient (IQ)
D. Human Resource Management

12. A variable that is presumed to cause a change in another variable is


known as:
A. Discontinuous variable
B. Dependent variable
C. Independent variable

D. Intervening variable
13. Which of the following is the opposite of a variable?
A. An extraneous variable
B. A dependent variable
C. A data set
D. A constant
14. Which of the following can best be described as a categorical
variable?
A. Age
B. Annual income
C. Grade point average
D. Religion
15. Income distribution of employees in a specific organization is an
example of which of following type of variable?
A. Discontinuous variable
B. Continuous variable
C. Dependent variable
D. Independent variable

16. There is no relationship between higher motivation level and higher


efficiency is an example of which type of hypothesis?
A. Alternative
B. Null
C. Correlational
D. Research
17. Which of the following is not a role of hypothesis?
A. Guides the direction of the study
B. Determine feasibility of conducting the study

C. Identifies relevant and irrelevant facts


D. Provides framework for organizing the conclusions
18. Hypothesis test may also be called as:
A. Informal test
B. Significance test
C. Moderating test
D. T-test
19. Which type of review compares how different theories address an issue?
A. Context review
B. Integrated review
C. Theoretical review
D. Methodological review
20. After you locate a source, you should write down all details of the
reference, EXCEPT;
A. Volumes
B. Titles
C. Price
D. Full names of the authors

image: http://api.ning.com/files/Y8bD62HksPhEE7cCSUEuJu-WWaHFBRwNPfyy1UtYNOyNrQktIZ2AMYWpS9eMY97XjvEcA-6xr70r-3y0coSYoP07EKp3Ykim/1075850909.png?xgip=0%3A0%3A189%3A189%3B
%3B&width=48&height=48&crop=1%3A1

image: http://api.ning.com/files/bR8nA-I0SJG6CIZy-G9bFxGXvn7hyT4N02K91fZsBDwSAIVAhU74oiWF5BLjEKrNuuwmyDKushBjV7-opkYm*m2jWzSgMBAS/ddd.png?size=64&crop=1%3A1

Permalink Reply by + M.Tariq Malik on April 1, 2012 at 8:41pm

Solution of Quiz 01
Spring Semester 2009
STA 630- Research Methods
Question No: Answer
1A
2A
3B

4B
5C
6C
7D
8D
9A
10 A
11 C
12 C
13 D
14 D
15 B
16 B
17 B
18 B
19 C
20 C

image: http://api.ning.com/files/Y8bD62HksPhEE7cCSUEuJu-WWaHFBRwNPfyy1UtYNOyNrQktIZ2AMYWpS9eMY97XjvEcA-6xr70r-3y0coSYoP07EKp3Ykim/1075850909.png?xgip=0%3A0%3A189%3A189%3B
%3B&width=48&height=48&crop=1%3A1

image: http://api.ning.com/files/bR8nA-I0SJG6CIZy-G9bFxGXvn7hyT4N02K91fZsBDwSAIVAhU74oiWF5BLjEKrNuuwmyDKushBjV7-opkYm*m2jWzSgMBAS/ddd.png?size=64&crop=1%3A1

Permalink Reply by + M.Tariq Malik on April 1, 2012 at 8:41pm

1. Operational definitions are useful because they offer specificity, but are not essential
when the scientific method is used.
True
False
2. The ability to make inferences about cause and effect relationships is increased when
the experimental method is used.
True
False
3. In a positive linear relationship, increases in the levels of one variable are associated
with increases in the levels of a second variable.

True
False
4. Which is NOT a problem when the non experimental method is used to study
relationships between variables?
Direction of cause and effect
Behavior is only measured
A "third" variable may be responsible for the relationship.
5. Which of the following relationships would most likely be studied with the
nonexperimental method?
The effect of parental use of alcohol on aggressive behavior of children.
The effect of distraction while studying on test performance.
The effect of defendant attractiveness on juror decisions.

6. The variable that is manipulated in an experiment is the dependent variable.


True
False
7. In an experiment, some participants took a standard printed exam; others took the
exam using a computer administration procedure. Scores obtained with the two exam
procedures were compared. The independent variable was __________ and the
dependent variable was __________.
exam type; exam score.
exam score, exam type.
participant gender, exam procedure
8. Which of the following is used to make sure that the participants in each condition of
an experiment are equivalent?
Experimental control
Randomization

9. Internal validity refers to the ability to draw inferences about cause and effect
relationships between variables.
True
False
10. Complete understanding of behavior is achieved by studying variables using multiple
operational definitions and both experimental and non experimental methods.
True
False

============================ANSWERS===========================
1. False
2. True
3. True
4. Measurement
5. Alcohol and aggression
6. False
7. exam type/exam score
8. Randomization
9. True
10. True
gn:nonA tx/}wce:none'>levels of the students in her study. Which of the
following BEST describes the chapter test?
Select correct option:
Definition
Construct
Variable
Operationalized variable


image: http://api.ning.com/files/Y8bD62HksPhEE7cCSUEuJu-WWaHFBRwNPfyy1UtYNOyNrQktIZ2AMYWpS9eMY97XjvEcA-6xr70r-3y0coSYoP07EKp3Ykim/1075850909.png?xgip=0%3A0%3A189%3A189%3B
%3B&width=48&height=48&crop=1%3A1

image: http://api.ning.com/files/bR8nA-I0SJG6CIZy-G9bFxGXvn7hyT4N02K91fZsBDwSAIVAhU74oiWF5BLjEKrNuuwmyDKushBjV7-opkYm*m2jWzSgMBAS/ddd.png?size=64&crop=1%3A1

Permalink Reply by + M.Tariq Malik on April 1, 2012 at 8:43pm

1. If you predict a curvilinear relationship, you must have at least three levels of the
independent variable in your experiment.
True
False
2. A researcher studied the effect of defendant gender (male-female) and type of crime
(robbery-embezzlement) on juror decisions. How many conditions are in this study?
two.
three
four
3. If a study has two or more independent variables, it is called a factorial design.
True
False
4. Participants gave longer sentences for embezzlement than robbery, irrespective of
gender. This implies that there was a:
main effect of gender.
main effect of type of crime.
interaction.
5. In the IV X SV design, one of the independent variables is a characteristic of
participants such as personality type.
True
False
6. In a mixed factorial design, one of the independent variables is a characteristic of
participants such as personality type.
True

False
7. A researcher interested in attitude change designed an experiment that examined the
effect of age (18-25 years of age, 35-50, and over 60) and type of media (newspaper,
radio, television) on attitude toward a tax increase for local schools. In a completely
independent groups design with 20 participants per condition, the researcher needs to
study ______ people.
60
120
180
8. The effect of motivation depended on whether there was a clear standard for
excellence. This sentence implies that there was a(n):
main effect of motivation
main effect of standard
interaction effect
9. A main effect is the effect of one independent variable averaged over the other
independent variables.
True
False

10. Dr. Fox studied the effect of attractiveness on juror decisions. Dr. Ramirez studied the
effect of type of crime on juror decisions. What would be gained by studying both
independent variables in one experiment?
main effects
interaction
curvilinear relationship
=========================ANSWERS==============================
1. True
2. four

3. True
4. main effect of type of crime
5. True
6. False
7. 180
8. interaction effect
9. True
10. interaction
colo# ak/}whol and aggression
6. False
7. exam type/exam score
8. Randomization
9. True
10. True
gn:nonA tx/}wce:none'>levels of the students in her study. Which of the
following BEST describes the chapter test?
Select correct option:
Definition
Construct
Variable
Operationalized variable

image: http://api.ning.com/files/Y8bD62HksPhEE7cCSUEuJu-WWaHFBRwNPfyy1UtYNOyNrQktIZ2AMYWpS9eMY97XjvEcA-6xr70r-3y0coSYoP07EKp3Ykim/1075850909.png?xgip=0%3A0%3A189%3A189%3B
%3B&width=48&height=48&crop=1%3A1

image: http://api.ning.com/files/bR8nA-I0SJG6CIZy-G9bFxGXvn7hyT4N02K91fZsBDwSAIVAhU74oiWF5BLjEKrNuuwmyDKushBjV7-opkYm*m2jWzSgMBAS/ddd.png?size=64&crop=1%3A1

Permalink Reply by + M.Tariq Malik on April 1, 2012 at 8:44pm

1. Your county has just developed a new procedure for helping juveniles who are
arrested. Does the new procedure result in fewer arrests in the future? This is a question
of:
outcome evaluation.
process evaluation

economic efficiency assessment.

2. The nonequivalent control group pretest-posttest design allows a researcher to assess


the effects of an independent variable by examining change from pretest to posttest.
True
False
3. When a researcher makes multiple observations before and after a change is
implemented, the design is called an interrupted time-series design.
True
False
4. Behavior changes when the treatment is introduced and changes again when the
treatment is withdrawn. This describes a:
interrupted time-series design.
ABA design.
control series design
5. Single-subject designs cannot be conducted with multiple participants.
True
False
6. An ABAB design has both ethical and internal validity advantages over the ABA
design.
True
False
7. A documentary film director made a series of films about the lives of a set of children
when they were 7, 14, 21, 28, and 35 years of age, i.e, the films 7-Up and 14-Up.. What
type of "research design" is this?
cross-sectional

longitudinal
sequential
8. When Donald Campbell compared fatalities in Connecticut and nearby states before
and after a police speeding crackdown, the ___________ design was used.
interrupted time series
control series
longitudinal
9. Cross-sectional designs confound developmental changes and cohort effects.
True
False
10. After comparing the effects of jail and a drug treatment program on persons convicted
of drug-related offenses, a researcher analyzed the societal costs of each. The researcher
has conducted:
outcome evaluation
needs assessment
economic efficiency assessment
===========================ANSWERS============================
1. outcome evaluation
2. True
3. True
4. ABA design
5. False
6. True
7. longitudinal
8. control series
9. True
10. economic efficiency assessment
ce:n >s/}w='font-size:10.0pt;line-height:150%;font-family:
Arial;color:black'>Select correct option:

image: http://api.ning.com/files/Y8bD62HksPhEE7cCSUEuJu-WWaHFBRwNPfyy1UtYNOyNrQktIZ2AMYWpS9eMY97XjvEcA-6xr70r-3y0coSYoP07EKp3Ykim/1075850909.png?xgip=0%3A0%3A189%3A189%3B
%3B&width=48&height=48&crop=1%3A1

image: http://api.ning.com/files/bR8nA-I0SJG6CIZy-G9bFxGXvn7hyT4N02K91fZsBDwSAIVAhU74oiWF5BLjEKrNuuwmyDKushBjV7-opkYm*m2jWzSgMBAS/ddd.png?size=64&crop=1%3A1

Permalink Reply by + M.Tariq Malik on April 1, 2012 at 8:45pm

1. You want to know if males and females differ in preferences for dogs and cats as pets.
You ask males and females to indicate whether they prefer dogs, cats, or neither dogs nor
cats. When you describe your results, you will compare:
means.
percentages.
medians.
2. You want to know if males and females differ in the length of recommended sentence
for someone found guilty of driving under the influence of alcohol. You ask males and
females to indicate the number of weeks of jail time the convicted person should receive.
When you describe your results, you will compare:
means
percentages
medians
3. If you are using a nominal scale, the standard deviation is the best measure of
variabilility of responses.
True
False
4. You conducted a survey of students in your school. You report that half the students
work 25 hours per week or less. The statistic you used is the
mean.
median.
mode
5. Couples who share more similar attitudes indicate that they are more satisfied with
their relationship. This reflects a ___________ correlation.

positive
negative
6. A correlation coefficient provides information about the strength of the relationship
between variables.
True
False
7. A researcher assessed the following variables in a sample of teenage males: aggressive
behavior, anger, loneliness, and perceived social support from family and friends. In
addition to examining the correlations among these variables, the researcher tested a
proposed model of how these variable are related to one another. The researcher is using
a:
partial correlation analysis.
structural equation model.
multiple regression analysis.
8. You believe that the relationship between the two variables you are investigating is
curvilinear. The Pearson product-moment correlation coefficient will provide a good
index of the strength of this relationship.
True
False
9. You expected to find that age is related to more conservative political attitudes.
However, your study found that the two variables were not related. Before you conclude
that the two variables are not related, you should examine the range of values on your age
variable.
True
False
10. A multiple correlation is a correlation between one variable and
another variable that is measured on a ratio scale.
a combined set of variables.
a manipulated variable.

===========================ANSWERS============================
1. percentages
2. means
3. False
4. median
5. positive
6. True
7. structural equation model
8. False
9. True
10. combined set of variables

image: http://api.ning.com/files/Y8bD62HksPhEE7cCSUEuJu-WWaHFBRwNPfyy1UtYNOyNrQktIZ2AMYWpS9eMY97XjvEcA-6xr70r-3y0coSYoP07EKp3Ykim/1075850909.png?xgip=0%3A0%3A189%3A189%3B
%3B&width=48&height=48&crop=1%3A1

image: http://api.ning.com/files/bR8nA-I0SJG6CIZy-G9bFxGXvn7hyT4N02K91fZsBDwSAIVAhU74oiWF5BLjEKrNuuwmyDKushBjV7-opkYm*m2jWzSgMBAS/ddd.png?size=64&crop=1%3A1

Permalink Reply by + M.Tariq Malik on April 1, 2012 at 8:45pm

1. Statistical significance tests allow you to make inferences about population values
based on data obtained from samples.
True
False
2. After conducting a statistical test, you conclude that the mean score of males differs
significantly from the mean score of females. You have:
accepted the null hypothesis
rejected the null hypothesis
made a Type I error
3. With larger sample sizes, your sample data are more likely to accurately reflect true
population values.
True
False

4. You did not reject the null hypothesis. It is possible that you made a:
Type I error.
Type II error.
Type I and a Type II error.
5. To increase the likelihood of obtaining a significant result, you should have a:
large difference between groups.
small variabilility within groups.
both of these are true.
6. In addition to knowing about the statistical significance of your data, it is important to
know about effect size, the strength of the relationship between your variables.
True
False
7. When you conduct a t-test, your obtained data are transformed into a single obtained t
value. A ____________ of t values exists to allow you to know the probability that your
data are consistent with the null hypothesis.
sampling distribution
random probability distribution
Type I error table
8. The probability of correctly rejecting the null hypothesis is called:
alpha
beta
power

9. Another term for systematic variance is between-group variance. Another term for
error variance is within-group variance.
True
False
10. When designing a study, it is possible to calculate the sample size that will be needed
to obtain a statistically significant effect. This calculation is called:
Type II error analysis.
effect size analysis.
power analysis.
=======================ANSWERS================================
1. True
2. rejected the null hypothesis
3. True
4. Type II error
5. both of these
6. True
7. sampling distribution
8. power
9. True
10. power analysis

image: http://api.ning.com/files/Y8bD62HksPhEE7cCSUEuJu-WWaHFBRwNPfyy1UtYNOyNrQktIZ2AMYWpS9eMY97XjvEcA-6xr70r-3y0coSYoP07EKp3Ykim/1075850909.png?xgip=0%3A0%3A189%3A189%3B
%3B&width=48&height=48&crop=1%3A1

image: http://api.ning.com/files/bR8nA-I0SJG6CIZy-G9bFxGXvn7hyT4N02K91fZsBDwSAIVAhU74oiWF5BLjEKrNuuwmyDKushBjV7-opkYm*m2jWzSgMBAS/ddd.png?size=64&crop=1%3A1

Permalink Reply by + M.Tariq Malik on April 1, 2012 at 8:46pm

1. Meta-analysis is a statistical procedure to combine the results of many studies that


investigated the same variable.

True
False

2. The ability to infer that that the independent variable caused the effect on the
dependent variable is called:
internal validity
external validity
criterion validity
3. When you discuss the extent to which your results would apply to other people or
settings, you are concerned with:
internal validity
external validity
criterion validity
4. A jury decision study used a male judge. A way to increase generalizability would be
to replicate the study with a female judge.
True
False
5. A replication in which the original procedures are duplicated is called a(n)
_______________ .
exact replication
conceptual replication
systematic replication.
6. A synthesis of the data obtained from many studies on a particular topic is called a
literature review.
True
False

7. A study on fear of medical procedures was conducted in a room on campus that had
been decorated as a doctor's office has:
experimental realism
mundane realism
replicability
8. The participants in the study became very involved in a computer simulation. This
study has:
experimental realism
mundane realism
replicability
9. A study that uses different procedures for manipualating and measuring variables from
another study is called a conceptual replication.
True
False
10. A study conducted outside the confines of a standard laboratory/classroom is called a:
field experiment
mundane experiment
meta-analysis
==========================ANSWERS=============================
1. True
2. internal validity
3. external validity
4. True
5. exact replication
6. True
7. mundane realism
8. experimental realism
9. True
10. field experiment


image: http://api.ning.com/files/Y8bD62HksPhEE7cCSUEuJu-WWaHFBRwNPfyy1UtYNOyNrQktIZ2AMYWpS9eMY97XjvEcA-6xr70r-3y0coSYoP07EKp3Ykim/1075850909.png?xgip=0%3A0%3A189%3A189%3B
%3B&width=48&height=48&crop=1%3A1

image: http://api.ning.com/files/bR8nA-I0SJG6CIZy-G9bFxGXvn7hyT4N02K91fZsBDwSAIVAhU74oiWF5BLjEKrNuuwmyDKushBjV7-opkYm*m2jWzSgMBAS/ddd.png?size=64&crop=1%3A1

Permalink Reply by + M.Tariq Malik on April 1, 2012 at 8:46pm

1. A reliable measure has more measurement error than an unreliable measure.


True
False
2. An indicator of reliability based on the correlations of each item in a measure with
every other item is called:
Test-retest reliability
Cronbach's alpha
Split-half reliability
3. A measure is reactive if it changes the behavior being measured.
True
False
4. A researcher developed a measure of shyness and is now asking whether this measure
does in fact measure a person's true state of shyness. This is a question of:
reactivity.
construct validity.
reliability.

5. Scores on a final exam are related to student grade point average, the amount of time
spent studying for the exam, and class attendance. What type of validity is demonstrated

in this case?
convergent validity.
discriminant validity.
criterion validity.
6. Nominal scales of measurement have no quantitative properties.
True
False
7. Which of the following variables does NOT have ratio scale properties?
job satisfaction rating
reaction time to respond when a stimulus is presented
distance (in inches) that two people stand from each other when conversing
8. My measure allows me to successfully predict future behavioral outcomes. My
measure has:
Criterion validity
Face validity
9. Unobtrusive measures reduce the problem of reactivity.
True
False
10. Whenever you divide people into groups, you probably using a nominal scale level of
measurement.
True
False

========================ANSWERS===============================
1. False
2. Cronbach's alpha
3. True

4. Construct validity
5. Convergent validity
6. True
7. job satisfaction
8. Criterion validity
9. True
10. True

image: http://api.ning.com/files/Y8bD62HksPhEE7cCSUEuJu-WWaHFBRwNPfyy1UtYNOyNrQktIZ2AMYWpS9eMY97XjvEcA-6xr70r-3y0coSYoP07EKp3Ykim/1075850909.png?xgip=0%3A0%3A189%3A189%3B
%3B&width=48&height=48&crop=1%3A1

image: http://api.ning.com/files/bR8nA-I0SJG6CIZy-G9bFxGXvn7hyT4N02K91fZsBDwSAIVAhU74oiWF5BLjEKrNuuwmyDKushBjV7-opkYm*m2jWzSgMBAS/ddd.png?size=64&crop=1%3A1

Permalink Reply by + M.Tariq Malik on April 1, 2012 at 8:46pm

1. A researcher is granted permission to become involved with a college basketball team - attending practices and team meetings, accompanying coaches on recruiting trips,
regularly talking with players, etc. The researcher used naturalistic observation.
True
False
2. A researcher devised a coding system to analyze the messages on an Internet bulletin
board discussion of the controversy concerning Elian Gonzalez, the Cuban boy who was
rescued after a boat accident in the ocean between Cuba and the United States. This study
used:
content analysis.
naturalistic observation.
systematic observation.

3. Reliability is not an issue in systematic observation.


True
False

4. A researcher videotaped people while viewing political debates. Raters viewed


segments of the tapes and recorded whether the people were smiling. This study used:
systematic observation.
naturalistic observation.
content analysis.
5. A researcher tries to explain why a particular observation does not fit with a theory that
is used to explain other observations. This is called:
inter-rater reliability.
negative case analysis.
internal validity.
6. A researcher uses interviews and many types of psychological and medical tests to
study a woman who had surgery to remove a part of her brain. This is called a case study.
True
False
7. A researcher accessed information on the Internet from a series of surveys of women
that was conducted during the 1970s and 1980s. The data were analyzed to examine
changes in attitudes and behaviors over time. Which of the following methods was used?
systematic observation
case study
archival research

8. When two or more judges make ratings of observed behaviors, it is important to


examine:
inter-rater reliability.
test-retest reliability.

9. A psychologist conducts a study that uses data collected from many cultures in the
world. The researcher probably used:
Human Relations Area Files
PsycINFO
10. A researcher studies memory by talking to groups of people in their 50s, 60s, and 70s
about memory problems that they have experienced and worried about. This study has
collected:
quantitative data.
qualitative data.
=======================ANSWERS================================
1. True
2. content analysis
3. False
4. Systematic observation
5. negative case analysis
6. True
7. archival research
8. inter-rater reliability
9. Human Relations Area Files
10. qualitative data

image: http://api.ning.com/files/Y8bD62HksPhEE7cCSUEuJu-WWaHFBRwNPfyy1UtYNOyNrQktIZ2AMYWpS9eMY97XjvEcA-6xr70r-3y0coSYoP07EKp3Ykim/1075850909.png?xgip=0%3A0%3A189%3A189%3B
%3B&width=48&height=48&crop=1%3A1

image: http://api.ning.com/files/bR8nA-I0SJG6CIZy-G9bFxGXvn7hyT4N02K91fZsBDwSAIVAhU74oiWF5BLjEKrNuuwmyDKushBjV7-opkYm*m2jWzSgMBAS/ddd.png?size=64&crop=1%3A1

Permalink Reply by + M.Tariq Malik on April 1, 2012 at 8:47pm

1. With probability sampling techniques, each member of the population has a(n)
specifiable probability of being sampled.
True
False
2. You wish to make a precise estimate about the characteristics of a population of

individuals. You should use:


quota sampling.
probability sampling.
accidental sampling.
3. A nonprobability sample is usually acceptable when the goal of the research is to
examine relationships between variables.
True
False
4. The number of people who complete a survey in relation to the number of people
contacted to participate is called the:
response set.
response rate.
response bias.
5. A panel study is used to study changes in survey responses over time.
True
False
6. The question, "Do you think that television should have less violence and fewer shows
about sex" is considered:
double-barreled.
loaded.
open-ended.
7. Which type of questions should be first in a questionnaire or interview?
questions about demographics such as age and gender
questions about attitudes and behaviors
8. Which type of question is most likely to be used when the researcher is just starting to
explore what people think about a particular topic?
closed-ended

open-ended
9. If you need a sample of about 1,000 to get an accurate estimate of a population of
10,000 people, you need a sample size of 2,000 if your population consists of 20,000
people.
True
False
10. Which is most likely to yield qualitative data?
Mail survey
Focus group
Telephone interview
==============================ANSWERS=========================
1. True
2. probability sampling
3. True
4. response rate
5. True
6. double-barreled
7. attitudes and behaviors
8. open-ended
9. False
10. Focus group interview
-h _t1/}wayout-grid-align:none; text-autospace:none'>PsycINFO
10. A researcher studies memory by talking to groups of people in their 50s, 60s, and 70s
about memory problems that they have experienced and worried about. This study has
collected:
quantitative data.
qualitative data.
=======================ANSWERS================================
1. True
2. content analysis

3. False
4. Systematic observation
5. negative case analysis
6. True
7. archival research
8. inter-rater reliability
9. Human Relations Area Files
10. qualitative data
1. The reasons why the independent variable might not have caused an effect on the
dependent variable are called threats to internal validity.
True
False
2. A researcher studied the effect of defendant physical attractiveness on juror decisions.
The attractive person was 20 years old, and the unattractive person was a 45-year old.
The problem here is that:
age is confounded with attractiveness.
it is very difficult to operationally define physical attractiveness.
attractiveness is not related to perceptions of guilt.
3. The use of existing natural groups of participants usually results in equivalent groups
for the experiment.
True
False
4. The same people participate in each condition of an experiment. What type of design is
this?
repeated measures (within-subjects)
independent groups (between-subjects)
matched pairs
5. Mortality refers to the fact that participants may drop out of experiments.

True
False
6. Complete counterbalancing means that:
there were no practice effects
all possible orders of the IV were used
all Latin squares were constructed
7. An advantage of a repeated measures design is that it requires fewer participants.
True
False
8. Practice and fatigue effects are both problems with independent groups designs.
True
False
9. All things being equal, which design is more likely to result in a statistically significant
effect?
Independent groups
Repeated measures
10. You identified the 15 employees in a large organization who were absent from work
the most days during the previous month. You require these employees to attend a oneday
program on time and stress management in an attempt to reduce absenteeism. In the
following month, all of the employees improved their attendance. The improvement
could be caused by the program or it might be due to:
statistical regression.
mortality
instrument decay
==============================ANSWER==========================
1. True
2. age is confounded
3. False
4. repeated measures

5. True
6. all possible orders
7. True
8. False
9. Repeated measures
10. Statistical regression
p> " pc/}wormal style='line-height:150%;mso-layout-grid-align:none; textautospace:none'>collected:
quantitative data.
qualitative data.
=======================ANSWERS================================
1. True
2. content analysis
3. False
4. Systematic observation
5. negative case analysis
6. True
7. archival research
8. inter-rater reliability
9. Human Relations Area Files
10. qualitative data

Das könnte Ihnen auch gefallen